You are on page 1of 63

10/30/23, 02:04 10/30/23, 02:04

! ! ! ! !!! ! !!! !! ! ! ! ! ! ! !
! !!!!!! !!! ! !!! ! ! ! ! !! ! ! !!! ! !! ! ! !!!

!! ! ! ! !! ! !! ! ! ! ! ! !! ! ! ! !!! ! ! ! ! !!!!! !
!! !! ! ! !! ! !!! !
! ! ! !! ! !! !

ID: 948087f2

Which of the following ordered pairs (x, y) satisfies the


system of inequalities above?

A.

B.

C.

D.

ID: 948087f2 Answer


Correct Answer: D

Rationale

Choice D is correct. Any point (x, y) that is a solution to the given system of inequalities must satisfy both
inequalities in the system. The second inequality in the system can be rewritten as . Of the given
answer choices, only choice D satisfies this inequality, because inequality is a true statement. The
point also satisfies the first inequality.

Alternate approach: Substituting into the first inequality gives , or , which is a true
statement. Substituting into the second inequality gives , or , which is a true
statement. Therefore, since satisfies both inequalities, it is a solution to the system.

Choice A is incorrect because substituting −2 for x and −1 for y in the first inequality gives , or
, which is false. Choice B is incorrect because substituting −1 for x and 3 for y in the first inequality

about:srcdoc Page 1 of 126 about:srcdoc Page 2 of 126


10/30/23, 02:04 10/30/23, 02:04

gives , or , which is false. Choice C is incorrect because substituting 1 for x and 5 for y in

the first inequality gives , or , which is false. ! ! ! ! !!! ! !!! !! ! ! ! ! ! ! !


! !!!!!! !!! ! !!! ! ! ! ! !! ! ! !!! ! !! ! ! !!!
Question Difficulty: Medium
!! ! ! ! !! ! !! ! ! ! ! ! !! ! ! ! !! ! ! ! !!! ! ! !!!
! ! ! !! ! ! !! ! !!

ID: 8c515062

A candle is made of ounces of wax. When the candle is burning, the amount of wax in the candle decreases by
ounce every hours. If ounces of wax remain in this candle, for how many hours has it been burning?

A.

B.

C.

D.

ID: 8c515062 Answer


Correct Answer: D

Rationale

Choice D is correct. It’s given that the candle starts with 17 ounces of wax and has 6 ounces of wax remaining
after a period of time has passed. The amount of wax the candle has lost during the time period can be found

17 − 6 ounces, or 11 ounces. This means the candle loses 11 ounces of wax during that period of time. It’s given
by subtracting the remaining amount of wax from the amount of wax the candle was made of, which yields

that the amount of wax decreases by 1 ounce every 4 hours. If ℎ represents the number of hours the candle has
1 11
been burning, it follows that = . Multiplying both sides of this equation by 4ℎ yields ℎ = 44. Therefore, the
4 ℎ
candle has been burning for 44 hours.

1 ℎ 1 11
= rather than =
4 11 4 ℎ
Choice A is incorrect and may result from using the equation to represent the
situation, and then rounding to the nearest whole number.

Choice B is incorrect. This is the amount of wax, in ounces, remaining in the candle, not the number of hours it
has been burning.

1 6 1 11
= rather than =
4 ℎ 4 ℎ
Choice C is incorrect and may result from using the equation to represent the
situation.

Question Difficulty: Medium

about:srcdoc Page 3 of 126 about:srcdoc Page 4 of 126


10/30/23, 02:04 10/30/23, 02:04

! ! ! ! !!! ! !!! !! ! ! ! ! ! ! ! ! ! ! ! !!! ! !!! !! ! ! ! ! ! ! !


! !!!!!! !!! ! !!! ! ! ! ! !! ! ! !!! ! !! ! ! !!! ! !!!!!! !!! ! !!! ! ! ! ! !! ! ! !!! ! !! ! ! !!!

!! ! ! ! !! ! !! ! ! ! ! ! !! ! ! ! !! ! ! ! !!! ! ! !! ! ! ! !! ! !! ! ! ! ! ! ! ! !! ! ! !! ! !!! !
!!! ! ! ! !! ! ! ! !!! ! ! !!!
!! ! !! ! ! !! ! !! !
ID: 41fdc0b8

ID: e53688cb

The table above shows the population of Greenleaf, Idaho, for the years 2000 and 2010. If the relationship The solution to the given system of equations is . What is the value of ?
between population and year is linear, which of the following functions P models the population of Greenleaf t
years after 2000?
ID: e53688cb Answer
A.
Correct Answer: 18

B. Rationale

The correct answer is 18. It's given by the second equation in the system that 3𝑦 = 11. Substituting 11 for 3𝑦 in
the first equation in the system, 𝑥 + 3𝑦 = 29, yields 𝑥 + 11 = 29. Subtracting 11 from both sides of this equation
C.

D. yields 𝑥 = 18.

Question Difficulty: Medium

ID: 41fdc0b8 Answer


Correct Answer: A

Rationale

Choice A is correct. It is given that the relationship between population and year is linear; therefore, the
function that models the population t years after 2000 is of the form , where m is the slope and b
is the population when . In the year 2000, . Therefore, . The slope is given by

. Therefore, , which is equivalent to the


equation in choice A.

Choice B is incorrect and may be the result of incorrectly calculating the slope as just the change in the value
of P. Choice C is incorrect and may be the result of the same error as in choice B, in addition to incorrectly
using t to represent the year, instead of the number of years after 2000. Choice D is incorrect and may be the
result of incorrectly using t to represent the year instead of the number of years after 2000.

Question Difficulty: Medium

about:srcdoc Page 5 of 126 about:srcdoc Page 6 of 126


10/30/23, 02:04 10/30/23, 02:04

Since the point (0, 1) represents the y-intercept, it follows that 𝑏 = 1. Substituting −4 for 𝑚 and 1 for 𝑏 in the
equation 𝑦 = 𝑏 + 𝑚𝑥 yields 𝑦 = 1 − 4𝑥 as the equation of the boundary line. Since the shaded region represents
! ! ! ! !!! ! !!! !! ! ! ! ! ! ! !
inequality 𝑦 > 1 − 4𝑥.
all the points above this boundary line, it follows that the shaded region shown represents the solutions to the

! !!!!!! !!! ! !!! ! ! ! ! !! ! ! !!! ! !! ! ! !!!


Choice A is incorrect. This inequality represents a region below, not above, a boundary line with a slope of 4,
!! ! ! ! !! ! !! ! ! ! ! ! !! ! ! ! !!! ! ! ! ! !!!!! ! not −4.
!! !! ! ! !! ! !!! !
! ! ! !! ! !! ! Choice B is incorrect. This inequality represents a region below, not above, the boundary line shown.

Choice C is incorrect. This inequality represents a region whose boundary line has a slope of 4, not −4.
ID: d02193fb
Question Difficulty: Medium

The shaded region shown represents the solutions to which inequality?

A.

B.

C.

D.

ID: d02193fb Answer


Correct Answer: D

Rationale

slope-intercept form 𝑦 = 𝑏 + 𝑚𝑥, where 𝑚 is the slope and (0, 𝑏) is the y-intercept of the line. For the graph
Choice D is correct. The equation for the line representing the boundary of the shaded region can be written in

shown, the boundary line passes through the points (0, 1) and (1, − 3). Given two points on a line, (𝑥1 , 𝑦1 ) and
𝑦 − 𝑦1
(𝑥2 , 𝑦2 ), the slope of the line can be calculated using the equation 𝑚 = 2 . Substituting the points (0, 1) and
𝑥2 − 𝑥1
−3 − 1 −4
(1, − 3) for (𝑥1 , 𝑦1 ) and (𝑥2 , 𝑦2 ) in this equation yields 𝑚 = , which is equivalent to 𝑚 = , or 𝑚 = − 4.
1−0 1

about:srcdoc Page 7 of 126 about:srcdoc Page 8 of 126


10/30/23, 02:04 10/30/23, 02:04

! ! ! ! !!! ! !!! !! ! ! ! ! ! ! ! ! ! ! ! !!! ! !!! !! ! ! ! ! ! ! !


! !!!!!! !!! ! !!! ! ! ! ! !! ! ! !!! ! !! ! ! !!! ! !!!!!! !!! ! !!! ! ! ! ! !! ! ! !!! ! !! ! ! !!!

!! ! ! ! !! ! !! ! ! ! ! ! !! ! ! ! !! ! ! ! !!! ! ! !! ! ! ! !! ! !! ! ! ! ! ! !! ! ! ! !! ! ! ! !!! ! !

ID: 8a6de407 ID: 3ce92ce8

The function f is defined by , where m and b are constants. If


For the given function , the graph of in the xy-plane is parallel to line . What is the slope of line ?
and , what is the value of m ?

ID: 3ce92ce8 Answer


ID: 8a6de407 Answer
Correct Answer: 2
Rationale Rationale

The correct answer is 2. The slope-intercept form of an equation for a line is , where m is the slope The correct answer is 2. It’s given that function 𝑓 is defined by 𝑓(𝑥) = 2𝑥 + 3. Therefore, the equation
representing the graph of 𝑦 = 𝑓(𝑥) in the xy-plane is 𝑦 = 2𝑥 + 3, and the graph is a line. For a linear equation in
the form 𝑦 = 𝑚𝑥 + 𝑏, 𝑚 represents the slope of the line. Since the value of 𝑚 in the equation 𝑦 = 2𝑥 + 3 is 2, the
and b is the y-coordinate of the y-intercept. Two ordered pairs, and , can be used to compute the

slope of the line defined by function 𝑓 is 2. It's given that line 𝑗 is parallel to the line defined by function 𝑓. The
slopes of parallel lines are equal. Therefore, the slope of line 𝑗 is also 2.
slope using the formula . It’s given that and ; therefore, the two ordered

pairs for this line are and . Substituting these values for and gives ,
Question Difficulty: Medium
or 2.

Question Difficulty: Medium

about:srcdoc Page 9 of 126 about:srcdoc Page 10 of 126


10/30/23, 02:04 10/30/23, 02:04

! ! ! ! !!! ! !!! !! ! ! ! ! ! ! ! ! ! ! ! !!! ! !!! !! ! ! ! ! ! ! !


! !!!!!! !!! ! !!! ! ! ! ! !! ! ! !!! ! !! ! ! !!! ! !!!!!! !!! ! !!! ! ! ! ! !! ! ! !!! ! !! ! ! !!!

!! ! ! ! !! ! !! ! ! ! ! ! !! ! ! ! !! ! ! ! !!! ! ! !!! !! ! ! ! !! ! !! ! ! ! ! ! !! ! ! ! !! ! ! ! !!! ! ! !!!


!! ! !! ! ! !! ! !! ! ! ! ! !! ! ! !! ! !!

ID: fb43b85f ID: eafdbbbd

A line passes through the points and in the xy-plane. What is the slope of the line?
What value of is the solution to the given equation?

ID: fb43b85f Answer A.

Correct Answer: 1.636, 18/11 B.


Rationale

18
C.
. For a line that passes through the points (𝑥1 , 𝑦1 ) and (𝑥2 , 𝑦2 ) in the xy-plane, the slope
11 𝑦 − 𝑦1
The correct answer is
of the line can be calculated using the slope formula, 𝑚 = 2
𝑥2 − 𝑥1
D.
. It's given that a line passes through the
points (4, 6) and (15, 24) in the xy-plane. Substituting (4, 6) for (𝑥1 , 𝑦1 ) and (15, 24) for (𝑥2 , 𝑦2 ) in the slope formula,
𝑦 − 𝑦1 24 − 6 18 18
𝑚= 2 , yields 𝑚 = , or 𝑚 = . Therefore, the slope of the line is . Note that 18/11 and 1.636 are
𝑥2 − 𝑥1 15 − 4 11 11
ID: eafdbbbd Answer

examples of ways to enter a correct answer. Correct Answer: C

Rationale
Question Difficulty: Medium
Choice C is correct. For the given equation, (𝑥 + 5) is a factor of both terms on the left-hand side. Therefore, the
1 1 3 4
given equation can be rewritten as ( − )(𝑥 + 5) = − 7, or ( − )(𝑥 + 5) = − 7, which is equivalent to
4 3 12 12
1
− (𝑥 + 5) = − 7. Multiplying both sides of this equation by −12 yields 𝑥 + 5 = 84. Subtracting 5 from both sides
12
of this equation yields 𝑥 = 79.

7
Choice A is incorrect. This is the value of 𝑥 for which the left-hand side of the given equation equals , not −7.
12

Choice B is incorrect. This is the value of 𝑥 for which the left-hand side of the given equation equals 0, not −7.

209
Choice D is incorrect. This is the value of 𝑥 for which the left-hand side of the given equation equals −
12
, not
−7.

Question Difficulty: Medium

about:srcdoc Page 11 of 126 about:srcdoc Page 12 of 126


10/30/23, 02:04 10/30/23, 02:04

! ! ! ! !!! ! !!! !! ! ! ! ! ! ! ! ! ! ! ! !!! ! !!! !! ! ! ! ! ! ! !


! !!!!!! !!! ! !!! ! ! ! ! !! ! ! !!! ! !! ! ! !!! ! !!!!!! !!! ! !!! ! ! ! ! !! ! ! !!! ! !! ! ! !!!

!! ! ! ! !! ! !! ! ! ! ! ! !! ! ! ! !! ! ! ! !!! ! ! !!! !! ! ! ! !! ! !! ! ! ! ! ! !! ! ! ! !! ! ! ! !!! ! ! !!!


! ! ! !! ! ! !! ! !! !! ! !! ! ! !! ! !! !

ID: 45bba652 ID: 9ed4c1a2

If , what is What is the slope of the graph of in the xy-plane?

the value of ?
ID: 9ed4c1a2 Answer
A. 2
Correct Answer: 13.75, 55/4
B. 5
Rationale

55
. In the xy-plane, the graph of an equation in the form 𝑦 = 𝑚𝑥 + 𝑏, where 𝑚 and 𝑏 are
4
C. 7
The correct answer is
constants, has a slope of 𝑚 and a y-intercept of (0, 𝑏). Applying the distributive property to the right-hand side
27 15 55 15
of the given equation yields 𝑦 = 𝑥 + + 7𝑥. Combining like terms yields 𝑦 = 𝑥 + . This equation is in the
D. 12

4 4 4 4
55 15 1
form 𝑦 = 𝑚𝑥 + 𝑏, where 𝑚 = and 𝑏 = . It follows that the slope of the graph of 𝑦 = (27𝑥 + 15) + 7𝑥 in the xy-
4 4 4
ID: 45bba652 Answer 55
4
plane is . Note that 55/4 and 13.75 are examples of ways to enter a correct answer.
Correct Answer: A

Rationale Question Difficulty: Medium

Choice A is correct. Adding the like terms on the left-hand side of the given equation yields .
Dividing both sides of this equation by 5 yields .

Choice B is incorrect and may result from subtracting 5, not dividing by 5, on both sides of the equation
. Choice C is incorrect. This is the value of x, not the value of . Choice D is incorrect. This is

the value of , not the value of .

Question Difficulty: Medium

about:srcdoc Page 13 of 126 about:srcdoc Page 14 of 126


10/30/23, 02:04 10/30/23, 02:04

! ! ! ! !!! ! !!! !! ! ! ! ! ! ! ! ! ! ! ! !!! ! !!! !! ! ! ! ! ! ! !


! !!!!!! !!! ! !!! ! ! ! ! !! ! ! !!! ! !! ! ! !!! ! !!!!!! !!! ! !!! ! ! ! ! !! ! ! !!! ! !! ! ! !!!

!! ! ! ! !! ! !! ! ! ! ! ! ! ! !! ! ! !! ! !!! ! !! ! ! ! !! ! !! ! ! ! ! ! !! ! ! ! !! ! ! ! !!! ! !
!!! ! ! ! !! ! ! ! !!! ! ! !!!
!! ! !! ! ! !! ! !! !
ID: f0773a55

ID: b544a348

In the solution to the system of equations


above, what is the value of x ?

ID: b544a348 Answer

Rationale

The correct answer is 7. Subtracting the second equation from the first equation eliminates the variable y.

Dividing both sides of the resulting equation by 4 yields x = 7.


The graph represents the total charge, in dollars, by an electrician for hours of work. The electrician charges a
Question Difficulty: Medium onetime fee plus an hourly rate. What is the best interpretation of the slope of the graph?

A. The electrician’s hourly rate

B. The electrician’s onetime fee

C. The maximum amount that the electrician charges

D. The total amount that the electrician charges

ID: f0773a55 Answer


Correct Answer: A

Rationale

the graph represents the total charge, in dollars, for 𝑥 hours of work. This graph shows a linear relationship in
Choice A is correct. It’s given that the electrician charges a onetime fee plus an hourly rate. It's also given that

the xy-plane. Thus, the total charge 𝑦, in dollars, for 𝑥 hours of work can be represented as 𝑦 = 𝑚𝑥 + 𝑏, where 𝑚
is the slope and (0, 𝑏) is the y-intercept of the graph of the equation in the xy-plane. Since the given graph
represents the total charge, in dollars, by an electrician for 𝑥 hours of work, it follows that its slope is 𝑚, or the

about:srcdoc Page 15 of 126 about:srcdoc Page 16 of 126


10/30/23, 02:04 10/30/23, 02:04

electrician’s hourly rate.

Choice B is incorrect. The electrician's onetime fee is represented by the y-coordinate of the y-intercept, not
! ! ! ! !!! ! !!! !! ! ! ! ! ! ! !
the slope, of the graph.
! !!!!!! !!! ! !!! ! ! ! ! !! ! ! !!! ! !! ! ! !!!
Choice C is incorrect and may result from conceptual errors.
!! ! ! ! !! ! !! ! ! ! ! ! !! ! ! ! !! ! ! ! !!! ! !
Choice D is incorrect and may result from conceptual errors.

Question Difficulty: Medium ID: 441558e7

Scientists collected fallen acorns that each housed a colony of the ant species P. ohioensis and analyzed each
colony's structure. For any of these colonies, if the colony has worker ants, the equation ,
where , gives the predicted number of larvae, , in the colony. If one of these colonies has
worker ants, which of the following is closest to the predicted number of larvae in the colony?

A.

B.

C.

D.

ID: 441558e7 Answer


Correct Answer: A

Rationale

Choice A is correct. It's given that the equation 𝑦 = 0.67𝑥 + 2.6, where 20 ≤ 𝑥 ≤ 110, gives the predicted number
of larvae, 𝑦, in a colony of ants if the colony has 𝑥 worker ants. If one of these colonies has 58 worker ants, the
predicted number of larvae in that colony can be found by substituting 58 for 𝑥 in the given equation.
Substituting 58 for 𝑥 in the given equation yields 𝑦 = 0.67(58) + 2.6, or 𝑦 = 41.46. Of the given choices, 41 is
closest to the predicted number of larvae in the colony.

Choice B is incorrect. This is closest to the predicted number of larvae in a colony with 87 worker ants.

a colony is 58.
Choice C is incorrect. This is closest to the number of worker ants for which the predicted number of larvae in

Choice D is incorrect. This is closest to the predicted number of larvae in a colony with 280 worker ants.

Question Difficulty: Medium

about:srcdoc Page 17 of 126 about:srcdoc Page 18 of 126


10/30/23, 02:04 10/30/23, 02:04

! ! ! ! !!! ! !!! !! ! ! ! ! ! ! ! ! ! ! ! !!! ! !!! !! ! ! ! ! ! ! !


! !!!!!! !!! ! !!! ! ! ! ! !! ! ! !!! ! !! ! ! !!! ! !!!!!! !!! ! !!! ! ! ! ! !! ! ! !!! ! !! ! ! !!!

!! ! ! ! !! ! !! ! ! ! ! ! !! ! ! ! !! ! ! ! !!! ! ! !!! !! ! ! ! !! ! !! ! ! ! ! ! !! ! ! ! !! ! ! ! !!! ! ! !!!


!! ! !! ! ! !! ! !! ! ! ! ! !! ! ! !! ! !!

ID: 038d87d7 ID: 5ad9eff0

A neighborhood consists of a -hectare park and a -hectare residential area. The total number of trees in the The width of a rectangular dance floor is w feet. The length of the floor is 6
neighborhood is . The equation represents this situation. Which of the following is the feet longer than its width. Which of the following expresses the perimeter,
best interpretation of x in this context? in feet, of the dance floor in terms of w ?

A. The average number of trees per hectare in the park A.

B. The average number of trees per hectare in the residential area B.

C. The total number of trees in the park


C.
D. The total number of trees in the residential area
D.

ID: 038d87d7 Answer


Correct Answer: A ID: 5ad9eff0 Answer
Correct Answer: B
Rationale

Choice A is correct. It's given that a neighborhood consists of a 2-hectare park and a 35-hectare residential
Rationale

area and that the total number of trees in the neighborhood is 3,934. It's also given that the equation
2𝑥 + 35𝑦 = 3,934 represents this situation. Since the total number of trees for a given area can be determined by
Choice B is correct. It is given that the width of the dance #oor is w feet. The length is 6 feet longer than the
width; therefore, the length of the dance #oor is . So the perimeter is .

2𝑥 and 35𝑦 correspond to the number of trees in the park and in the residential area, respectively. Since 2𝑥
taking the number of hectares times the average number of trees per hectare, this must mean that the terms

corresponds to the number of trees in the park, and 2 is the size of the park, in hectares, 𝑥 must represent the
Choice A is incorrect because it is the sum of one length and one width, which is only half the perimeter.
Choice C is incorrect and may result from using the formula for the area instead of the formula for the
average number of trees per hectare in the park. perimeter and making a calculation error. Choice D is incorrect because this is the area, not the perimeter, of
the dance #oor.
Choice B is incorrect and may result from conceptual errors.
Question Difficulty: Medium
Choice C is incorrect and may result from conceptual errors.

Choice D is incorrect and may result from conceptual errors.

Question Difficulty: Medium

about:srcdoc Page 19 of 126 about:srcdoc Page 20 of 126


10/30/23, 02:04 10/30/23, 02:04

! ! ! ! !!! ! !!! !! ! ! ! ! ! ! ! ! ! ! ! !!! ! !!! !! ! ! ! ! ! ! !


! !!!!!! !!! ! !!! ! ! ! ! !! ! ! !!! ! !! ! ! !!! ! !!!!!! !!! ! !!! ! ! ! ! !! ! ! !!! ! !! ! ! !!!

!! ! ! ! !! ! !! ! ! ! ! ! !! ! ! ! !! ! ! ! !!! ! ! !!! !! ! ! ! !! ! !! ! ! ! ! ! !! ! ! ! !!! ! ! ! ! !!!!! !


!! ! !! ! ! !! ! !! ! !! !! ! ! !! ! !!! !
! ! ! !! ! !! !

ID: 6f6dfe3e
ID: 90bd9ef8

The average annual energy cost for a certain home is $4,334. The
homeowner plans to spend $25,000 to install a geothermal heating system.
The homeowner estimates that the average annual energy cost will then be
$2,712. Which of the following inequalities can be solved to find t, the
number of years after installation at which the total amount of energy cost
savings will exceed the installation cost?
The table shows three values of and their corresponding values of , where is a constant, for the linear
relationship between and . What is the slope of the line that represents this relationship in the xy-plane? A.

A.
B.
B.
C.
C.

D. D.

ID: 6f6dfe3e Answer ID: 90bd9ef8 Answer


Correct Answer: A Correct Answer: B
Rationale Rationale

Choice A is correct. The slope, 𝑚, of a line in the xy-plane can be found using two points on the line, (𝑥1 , 𝑦1 )
𝑦 − 𝑦1
and (𝑥2 , 𝑦2 ), and the slope formula 𝑚 = 2
Choice B is correct. The savings each year from installing the geothermal heating system will be the average

𝑥2 − 𝑥1
. Based on the given table, the line representing the relationship annual energy cost for the home before the geothermal heating system installation minus the average annual
between 𝑥 and 𝑦 in the xy-plane passes through the points (−6, 𝑛 + 184), (−3, 𝑛 + 92), and (0, 𝑛), where 𝑛 is a energy cost after the geothermal heating system installation, which is dollars. In t years, the
constant. Substituting two of these points, (−3, 𝑛 + 92) and (0, 𝑛), for (𝑥1 , 𝑦1 ) and (𝑥2 , 𝑦2 ), respectively, in the
𝑛 − (𝑛 + 92) 𝑛 − 𝑛 − 92 92
savings will be dollars. Therefore, the inequality that can be solved to find the number of
slope formula yields 𝑚 = , which is equivalent to 𝑚 = , or 𝑚 = − . Therefore, the slope of
0 − (−3) 0+3 3
years after installation at which the total amount of energy cost savings will exceed (be greater than) the
92
the line that represents this relationship in the xy-plane is − .
3
installation cost, $25,000, is .

Choice B is incorrect and may result from conceptual or calculation errors. Choice A is incorrect. It gives the number of years after installation at which the total amount of energy cost
savings will be less than the installation cost. Choice C is incorrect and may result from subtracting the
Choice C is incorrect and may result from conceptual or calculation errors. average annual energy cost for the home from the onetime cost
of the geothermal heating system installation. To find the predicted total savings, the predicted average cost
Choice D is incorrect and may result from conceptual or calculation errors. should be subtracted from the average annual energy cost before the installation, and the result should be
multiplied by the number of years, t. Choice D is incorrect and may result from misunderstanding the context.
Question Difficulty: Medium

The ratio compares the average energy cost before installation and the average energy cost after
about:srcdoc Page 21 of 126 about:srcdoc Page 22 of 126
10/30/23, 02:04 10/30/23, 02:04

The ratio compares the average energy cost before installation and the average energy cost after

installation; it does not represent the savings.


! ! ! ! !!! ! !!! !! ! ! ! ! ! ! !
Question Difficulty: Medium
! !!!!!! !!! ! !!! ! ! ! ! !! ! ! !!! ! !! ! ! !!!

!! ! ! ! !! ! !! ! ! ! ! ! !! ! ! ! !!! ! ! ! ! !!!!! !
!! !! ! ! !! ! !!! !
! ! ! !! ! !! !

ID: 8f0c82e2

The minimum value of is less than times another number . Which inequality shows the possible values of
?

A.

B.

C.

D.

ID: 8f0c82e2 Answer


Correct Answer: B

Rationale

Choice B is correct. It’s given that the minimum value of 𝑥 is 12 less than 6 times another number 𝑛. Therefore,
the possible values of 𝑥 are all greater than or equal to the value of 12 less than 6 times 𝑛. The value of 6 times 𝑛
is given by the expression 6𝑛. The value of 12 less than 6𝑛 is given by the expression 6𝑛 − 12. Therefore, the
possible values of 𝑥 are all greater than or equal to 6𝑛 − 12. This can be shown by the inequality 𝑥 ≥ 6𝑛 − 12.

Choice A is incorrect. This inequality shows the possible values of 𝑥 if the maximum, not the minimum, value
of 𝑥 is 12 less than 6 times 𝑛.

Choice C is incorrect. This inequality shows the possible values of 𝑥 if the maximum, not the minimum, value
of 𝑥 is 6 times 𝑛 less than 12, not 12 less than 6 times 𝑛.

Choice D is incorrect. This inequality shows the possible values of 𝑥 if the minimum value of 𝑥 is 6 times 𝑛 less
than 12, not 12 less than 6 times 𝑛.

Question Difficulty: Medium

about:srcdoc Page 23 of 126 about:srcdoc Page 24 of 126


10/30/23, 02:04 10/30/23, 02:04

! ! ! ! !!! ! !!! !! ! ! ! ! ! ! ! ! ! ! ! !!! ! !!! !! ! ! ! ! ! ! !


! !!!!!! !!! ! !!! ! ! ! ! !! ! ! !!! ! !! ! ! !!! ! !!!!!! !!! ! !!! ! ! ! ! !! ! ! !!! ! !! ! ! !!!

!! ! ! ! !! ! !! ! ! ! ! ! ! ! !! ! ! !! ! !!! ! !! ! ! ! !! ! !! ! ! ! ! ! !! ! ! ! !!! ! ! ! ! !!!!! !


!!! ! ! ! !! ! ! ! !!! ! ! !!! !! !! ! ! !! ! !!! !
!! ! !! ! ! !! ! !! ! ! ! ! !! ! !! !

ID: 686b7cad ID: f02b4509

A proposal for a new library was included on an election ballot. A radio show stated that times as many people A moving truck can tow a trailer if the combined weight of the trailer and the boxes it contains is no more than
voted in favor of the proposal as people who voted against it. A social media post reported that more pounds. What is the maximum number of boxes this truck can tow in a trailer with a weight of pounds if
people voted in favor of the proposal than voted against it. Based on these data, how many people voted against each box weighs pounds?
the proposal?
A.
A.
B.
B.
C.
C.
D.
D.

ID: f02b4509 Answer


ID: 686b7cad Answer Correct Answer: A
Correct Answer: A
Rationale
Rationale

Choice A is correct. It's given that a radio show stated that 3 times as many people voted in favor of the boxes it contains is no more than 4,600 pounds. If the trailer has a weight of 500 pounds and each box weighs
Choice A is correct. It’s given that the truck can tow a trailer if the combined weight of the trailer and the

proposal as people who voted against it. Let 𝑥 represent the number of people who voted against the proposal. 120 pounds, the expression 500 + 120𝑏, where 𝑏 is the number of boxes, gives the combined weight of the trailer
It follows that 3𝑥 is the number of people who voted in favor of the proposal and 3𝑥 − 𝑥, or 2𝑥, is how many and the boxes. Since the combined weight must be no more than 4,600 pounds, the possible numbers of boxes
the truck can tow are given by the inequality 500 + 120𝑏 ≤ 4,600. Subtracting 500 from both sides of this
reported that 15,000 more people voted in favor of the proposal than voted against it. Thus, 2𝑥 = 15,000. Since 205
more people voted in favor of the proposal than voted against it. It's also given that a social media post
inequality yields 120𝑏 ≤ 4,100. Dividing both sides of this inequality by 120 yields 𝑏 ≤ , or 𝑏 is less than or
6
2𝑥 = 15,000, the value of 𝑥 must be half of 15,000, or 7,500. Therefore, 7,500 people voted against the proposal. equal to approximately 34.17. Since the number of boxes, 𝑏, must be a whole number, the maximum number of
boxes the truck can tow is the greatest whole number less than 34.17, which is 34.
Choice B is incorrect. This is how many more people voted in favor of the proposal than voted against it, not
the number of people who voted against the proposal. Choice B is incorrect. Towing the trailer and 35 boxes would yield a combined weight of 4,700 pounds, which is
greater than 4,600 pounds.
Choice C is incorrect. This is the number of people who voted in favor of the proposal, not the number of
people who voted against the proposal. Choice C is incorrect. Towing the trailer and 38 boxes would yield a combined weight of 5,060 pounds, which is
greater than 4,600 pounds.
Choice D is incorrect and may result from conceptual or calculation errors.
Choice D is incorrect. Towing the trailer and 39 boxes would yield a combined weight of 5,180 pounds, which is
greater than 4,600 pounds.
Question Difficulty: Medium

Question Difficulty: Medium

about:srcdoc Page 25 of 126 about:srcdoc Page 26 of 126


10/30/23, 02:04 10/30/23, 02:04

! ! ! ! !!! ! !!! !! ! ! ! ! ! ! !
intersect at the point . Therefore, the system of equations in choice A has the same solution as the given
system.

! !!!!!! !!! ! !!! ! ! ! ! !! ! ! !!! ! !! ! ! !!!


Choices B, C, and D are incorrect. If graphed in the xy-plane, these choices would intersect at the points ,
!! ! ! ! !! ! !! ! ! ! ! ! ! ! !! ! ! !! ! !!! !
!!! ! ! ! !! ! ! ! !!! ! ! !!! , and , respectively, not .
!! ! !! ! ! !! ! !! !

Question Difficulty: Medium


ID: 2704399f

Which of the following systems of equations has the same solution as the
system of equations graphed above?

A.

B.

C.

D.

ID: 2704399f Answer


Correct Answer: A

Rationale

Choice A is correct. The solution to a system of equations is the coordinates of the intersection point of the
graphs of the equations in the xy-plane. Based on the graph, the solution to the given system of equations is

best approximated as . In the xy-plane, the graph of is a horizontal line on which every y-

coordinate is 0, and the graph of is a vertical line on which every x-coordinate is . These graphs

about:srcdoc Page 27 of 126 about:srcdoc Page 28 of 126


10/30/23, 02:04 10/30/23, 02:04

! ! ! ! !!! ! !!! !! ! ! ! ! ! ! ! ! ! ! ! !!! ! !!! !! ! ! ! ! ! ! !


! !!!!!! !!! ! !!! ! ! ! ! !! ! ! !!! ! !! ! ! !!! ! !!!!!! !!! ! !!! ! ! ! ! !! ! ! !!! ! !! ! ! !!!

!! ! ! ! !! ! !! ! ! ! ! ! !! ! ! ! !! ! ! ! !!! ! ! !!! !! ! ! ! !! ! !! ! ! ! ! ! !! ! ! ! !!! ! ! ! ! !!!!! !


! ! ! !! ! ! !! ! !! !! !! ! ! !! ! !!! !
! ! ! !! ! !! !

ID: 5a7ab8e8
ID: e9ef0e6b

How many solutions does the given equation have? A model estimates that whales from the genus Eschrichtius travel to miles in the ocean each day during
their migration. Based on this model, which inequality represents the estimated total number of miles, , a whale
A. Exactly one from the genus Eschrichtius could travel in days of its migration?

B. Exactly two A.

C. Infinitely many B.

D. Zero C.

D.
ID: 5a7ab8e8 Answer
Correct Answer: C
ID: e9ef0e6b Answer
Rationale
Correct Answer: B

Choice C is correct. If the two sides of a linear equation are equivalent, then the equation is true for any value. Rationale

equation 66𝑥 = 66𝑥 are equivalent, the given equation has infinitely many solutions. Choice B is correct. It's given that the model estimates that whales from the genus Eschrichtius travel 72 to 77
If an equation is true for any value, it has infinitely many solutions. Since the two sides of the given linear

miles in the ocean each day during their migration. If one of these whales travels 72 miles each day for 16 days,
then the whale travels 72(16) miles total. If one of these whales travels 77 miles each day for 16 days, then the
whale travels 77(16) miles total. Therefore, the model estimates that in 16 days of its migration, a whale from
Choice A is incorrect and may result from conceptual or calculation errors.

Choice B is incorrect and may result from conceptual or calculation errors. the genus Eschrichtius could travel at least 72(16) and at most 77(16) miles total. Thus, the inequality
(72)(16) ≤ 𝑥 ≤ (77)(16) represents the estimated total number of miles, 𝑥, a whale from the genus
Eschrichtius could travel in 16 days of its migration.
Choice D is incorrect and may result from conceptual or calculation errors.

Question Difficulty: Medium


Choice A is incorrect and may result from conceptual errors.

Choice C is incorrect and may result from conceptual errors.

Choice D is incorrect and may result from conceptual errors.

Question Difficulty: Medium

about:srcdoc Page 29 of 126 about:srcdoc Page 30 of 126


10/30/23, 02:04 10/30/23, 02:04

! ! ! ! !!! ! !!! !! ! ! ! ! ! ! ! ! ! ! ! !!! ! !!! !! ! ! ! ! ! ! !


! !!!!!! !!! ! !!! ! ! ! ! !! ! ! !!! ! !! ! ! !!! ! !!!!!! !!! ! !!! ! ! ! ! !! ! ! !!! ! !! ! ! !!!

!! ! ! ! !! ! !! ! ! ! ! ! !! ! ! ! !! ! ! ! !!! ! ! !!! !! ! ! ! !! ! !! ! ! ! ! ! !! ! ! ! !!! ! ! ! ! !!!!! !


! ! ! !! ! ! !! ! !! !! !! ! ! !! ! !!! !
! ! ! !! ! !! !

ID: 36ab4122
ID: 74c98c82
Megan’s regular wage at her job is p dollars per hour for the first 8 hours of
work in a day plus 1.5 times her regular hourly wage for work in excess of An event planner is planning a party. It costs the event planner a onetime fee of to rent the venue and
8 hours that day. On a given day, Megan worked for 10 hours, and her total per attendee. The event planner has a budget of . What is the greatest number of attendees possible without
earnings for that day were $137.50. What is Megan’s regular hourly wage? exceeding the budget?

A. $11.75
ID: 74c98c82 Answer
B. $12.50
Correct Answer: 16
C. $13.25 Rationale

D. $13.75 The correct answer is 16. The total cost of the party is found by adding the onetime fee of the venue to the cost
per attendee times the number of attendees. Let 𝑥 be the number of attendees. The expression 35 + 10.25𝑥 thus
represents the total cost of the party. It's given that the budget is $ 200, so this situation can be represented by
the inequality 35 + 10.25𝑥 ≤ 200. The greatest number of attendees can be found by solving this inequality for 𝑥.
ID: 36ab4122 Answer
Subtracting 35 from both sides of this inequality gives 10.25𝑥 ≤ 165. Dividing both sides of this inequality by
10.25 results in approximately 𝑥 ≤ 16.098. Since the question is stated in terms of attendees, rounding 𝑥 down to
Rationale

Choice B is correct. Since p represents Megan’s regular pay per hour, 1.5p represents the pay per hour in excess the nearest whole number, 16, gives the greatest number of attendees possible.
of 8 hours. Since Megan worked for 10 hours, she must have been paid p dollars per hour for 8 of the hours plus
1.5p dollars per hour for the remaining 2 hours. Therefore, since Megan earned $137.50 for the 10 hours, the Question Difficulty: Medium
situation can be represented by the equation 137.5 = 8p + 2(1.5)p. Distributing the 2 in the equation gives 137.5 =
8p + 3p, and combining like terms gives 137.5 = 11p. Dividing both sides by 11 gives p = 12.5. Therefore, Megan’s
regular wage is $12.50.

Choices A and C are incorrect and may be the result of calculation errors. Choice D is incorrect and may result
from finding the average hourly wage that Megan earned for the 10 hours of work.

Question Difficulty: Medium

about:srcdoc Page 31 of 126 about:srcdoc Page 32 of 126


10/30/23, 02:04 10/30/23, 02:04

! ! ! ! !!! ! !!! !! ! ! ! ! ! ! ! ! ! ! ! !!! ! !!! !! ! ! ! ! ! ! !


! !!!!!! !!! ! !!! ! ! ! ! !! ! ! !!! ! !! ! ! !!! ! !!!!!! !!! ! !!! ! ! ! ! !! ! ! !!! ! !! ! ! !!!

!! ! ! ! !! ! !! ! ! ! ! ! !! ! ! ! !! ! ! ! !!! ! ! !!! !! ! ! ! !! ! !! ! ! ! ! ! ! ! !! ! ! !! ! !!! !


!! ! !! ! ! !! ! !! ! !!! ! ! ! !! ! ! ! !!! ! ! !!!
!! ! !! ! ! !! ! !! !

ID: 637022d2
ID: 5e422ff9

The given equation describes the relationship between the number of birds, , and the number of reptiles, , that
can be cared for at a pet care business on a given day. If the business cares for reptiles on a given day, how
many birds can it care for on this day?
In the solution to the system of equations
A. above, what is the value of y ?

A.
B.

C. B.

D. C.

D.
ID: 637022d2 Answer
Correct Answer: A
ID: 5e422ff9 Answer
Rationale

Choice A is correct. The number of birds can be found by calculating the value of 𝑏 when 𝑟 = 16 in the given
Correct Answer: D

equation. Substituting 16 for 𝑟 in the given equation yields 2.5𝑏 + 5(16) = 80, or 2.5𝑏 + 80 = 80. Subtracting 80
Rationale

from both sides of this equation yields 2.5𝑏 = 0. Dividing both sides of this equation by 2.5 yields 𝑏 = 0.
Therefore, if the business cares for 16 reptiles on a given day, it can care for 0 birds on this day.
Choice D is correct. Multiplying both sides of by 5 results in . Multiplying both sides
of by 2 results in . Subtracting the resulting equations yields ,
Choice B is incorrect and may result from conceptual or calculation errors. which simplifies to . Dividing both sides of by results in .

Choice C is incorrect and may result from conceptual or calculation errors.


Choices A and B are incorrect and may result from incorrectly subtracting the transformed equation. Choice C
Choice D is incorrect and may result from conceptual or calculation errors. is incorrect and may result from finding the value of x instead of the value of y.

Question Difficulty: Medium Question Difficulty: Medium

about:srcdoc Page 33 of 126 about:srcdoc Page 34 of 126


10/30/23, 02:04 10/30/23, 02:04

! ! ! ! !!! ! !!! !! ! ! ! ! ! ! ! ! ! ! ! !!! ! !!! !! ! ! ! ! ! ! !


! !!!!!! !!! ! !!! ! ! ! ! !! ! ! !!! ! !! ! ! !!! ! !!!!!! !!! ! !!! ! ! ! ! !! ! ! !!! ! !! ! ! !!!

!! ! ! ! !! ! !! ! ! ! ! ! !! ! ! ! !! ! ! ! !!! ! ! !!! !! ! ! ! !! ! !! ! ! ! ! ! !! ! ! ! !! ! ! ! !!! ! !


!! ! !! ! ! !! ! !! !

ID: 113b938e
ID: 2e1a7f66

Figure A and figure B are both regular polygons. The sum of the perimeter of figure A and the perimeter of figure
The equation above represents the speed y, in feet per second, of Sheila’s
B is inches. The equation represents this situation, where is the number of sides of figure A
bicycle x seconds after she applied the brakes at the end of a ride. If the
and is the number of sides of figure B. Which statement is the best interpretation of in this context?
equation is graphed in the xy-plane, which of the following is the best
A. Each side of figure B has a length of inches. interpretation of the x-coordinate of the line’s x-intercept in the context of
the problem?
B. The number of sides of figure B is .
A. The speed of Sheila’s bicycle, in feet per second, before Sheila applied the brakes
C. Each side of figure A has a length of inches.
B. The number of feet per second the speed of Sheila’s bicycle decreased each second after Sheila applied the
D. The number of sides of figure A is . brakes

C. The number of seconds it took from the time Sheila began applying the brakes until the bicycle came to a
ID: 2e1a7f66 Answer complete stop
Correct Answer: A
D. The number of feet Sheila’s bicycle traveled from the time she began applying the brakes until the bicycle
Rationale came to a complete stop

the perimeters of the two figures is 63 inches. It’s also given that 𝑥 is the number of sides of figure A and 𝑦 is
Choice A is correct. It’s given that figure A and figure B (not shown) are both regular polygons and the sum of

the number of sides of figure B, and that the equation 3𝑥 + 6𝑦 = 63 represents this situation. Thus, 3𝑥 and 6𝑦
ID: 113b938e Answer

represent the perimeters, in inches, of figure A and figure B, respectively. Since 6𝑦 represents the perimeter, in
Correct Answer: C

inches, of figure B and 𝑦 is the number of sides of figure B, it follows that each side of figure B has a length of Rationale
6 inches.
Choice C is correct. It’s given that for each point on the graph of the given equation, the x-coordinate
Choice B is incorrect. The number of sides of figure B is 𝑦, not 6. represents the number of seconds after Sheila applied the brakes, and the y-coordinate represents the speed of

Choice C is incorrect. Since the perimeter, in inches, of figure A is represented by 3𝑥, each side of figure A has a
Sheila’s bicycle at that moment in time. For the graph of the equation, the y-coordinate of the x-intercept is 0.

length of 3 inches, not 6 inches.


Therefore, the x-coordinate of the x-intercept of the graph of the given equation represents the number of
seconds it took from the time Sheila began applying the brakes until the bicycle came to a complete stop.

Choice D is incorrect. The number of sides of figure A is 𝑥, not 6.


Choice A is incorrect. The speed of Sheila’s bicycle before she applied the brakes is represented by the y-
coordinate of the y-intercept of the graph of the given equation, not the x-coordinate of the x-intercept. Choice
Question Difficulty: Medium
B is incorrect. The number of feet per second the speed of Sheila’s bicycle decreased each second after Sheila
applied the brakes is represented by the slope of the graph of the given equation, not the x-coordinate of the x-
intercept. Choice D is incorrect and may result from misinterpreting x as the distance, in feet, traveled after
applying the brakes, rather than the time, in seconds, after applying the brakes.

Question Difficulty: Medium

about:srcdoc Page 35 of 126 about:srcdoc Page 36 of 126


10/30/23, 02:04 10/30/23, 02:04

! ! ! ! !!! ! !!! !! ! ! ! ! ! ! ! ! ! ! ! !!! ! !!! !! ! ! ! ! ! ! !


! !!!!!! !!! ! !!! ! ! ! ! !! ! ! !!! ! !! ! ! !!! ! !!!!!! !!! ! !!! ! ! ! ! !! ! ! !!! ! !! ! ! !!!

!! ! ! ! !! ! !! ! ! ! ! ! ! ! !! ! ! !! ! !!! ! !! ! ! ! !! ! !! ! ! ! ! ! !! ! ! ! !! ! ! ! !!! ! !
!!! ! ! ! !! ! ! ! !!! ! ! !!!
!! ! !! ! ! !! ! !! !
ID: 1bc11c4e

ID: b3c7ca1d

The given function models the number of gallons of gasoline that remains from a full gas tank in a car after
Which of the following systems of linear equations has no solution?
driving miles. According to the model, about how many gallons of gasoline are used to drive each mile?
A.
A.

B.
B.
C.

C. D.

D. ID: 1bc11c4e Answer


Correct Answer: A

Rationale

Choice A is correct. It's given that the function 𝑔 models the number of gallons that remain from a full gas tank
ID: b3c7ca1d Answer

in a car after driving 𝑚 miles. In the given function 𝑔(𝑚) = − 0.05𝑚 + 12.1, the coefficient of 𝑚 is −0.05. This
Correct Answer: C

Rationale means that for every increase in the value of 𝑚 by 1, the value of 𝑔(𝑚) decreases by 0.05. It follows that for each
mile driven, there is a decrease of 0.05 gallons of gasoline. Therefore, 0.05 gallons of gasoline are used to drive
Choice C is correct. A system of two linear equations in two variables, 𝑥 and 𝑦, has no solution if the graphs of
each mile.

the xy-plane represented by equations in slope-intercept form, 𝑦 = 𝑚𝑥 + 𝑏, where 𝑚 and 𝑏 are constants, are
the lines represented by the equations in the xy-plane are distinct and parallel. The graphs of two lines in

parallel if their slopes, 𝑚, are the same and are distinct if their y-coordinates of the y-intercepts, 𝑏, are
Choice B is incorrect and represents the number of gallons of gasoline in a full gas tank.

different. In the equations 𝑦 = 16𝑥 + 3 and 𝑦 = 16𝑥 + 19, the values of 𝑚 are each 16, and the values of 𝑏 are 3 and
19, respectively. Since the slopes of these lines are the same, and the y-coordinates of the y-intercepts are
Choice C is incorrect and may result from conceptual errors.

different, it follows that the system of linear equations in choice C has no solution. Choice D is incorrect and may result from conceptual errors.

Question Difficulty: Medium


line. Therefore, this system has a solution, (3, 5), rather than no solution.
Choice A is incorrect. The lines represented by the equations in this system are a vertical line and a horizontal

coordinate of the y-intercept. Therefore, this system has a solution, (0, 6), rather than no solution.
Choice B is incorrect. The two lines represented by these equations have different slopes and the same y-

of 5 that have the same y-coordinate of the y-intercept. Therefore, this system has a solution, (0, 5), rather than
Choice D is incorrect. The two lines represented by these equations are a horizontal line and a line with a slope

no solution.

Question Difficulty: Medium

about:srcdoc Page 37 of 126 about:srcdoc Page 38 of 126


10/30/23, 02:04 10/30/23, 02:04

! ! ! ! !!! ! !!! !! ! ! ! ! ! ! ! ! ! ! ! !!! ! !!! !! ! ! ! ! ! ! !


! !!!!!! !!! ! !!! ! ! ! ! !! ! ! !!! ! !! ! ! !!! ! !!!!!! !!! ! !!! ! ! ! ! !! ! ! !!! ! !! ! ! !!!

!! ! ! ! !! ! !! ! ! ! ! ! !! ! ! ! !! ! ! ! !!! ! ! !! ! ! ! !! ! !! ! ! ! ! ! ! ! !! ! ! !! ! !!! !
!!! ! ! ! !! ! ! ! !!! ! ! !!!
!! ! !! ! ! !! ! !! !
ID: 042aa429

ID: e77a76ce
If and , what is the value of ?

A. Which of the following systems of linear equations has no solution?

B. A.

C.
B.
D.

C.
ID: 042aa429 Answer
Correct Answer: C
D.
Rationale

Choice C is correct. The value of 𝑓(2) can be found by substituting 2 for 𝑥 in the given equation 𝑓(𝑥) = 𝑥 + 7,
which yields 𝑓(2) = 2 + 7, or 𝑓(2) = 9. The value of 𝑔(2) can be found by substituting 2 for 𝑥 in the given
equation 𝑔(𝑥) = 7𝑥, which yields 𝑔(2) = 7(2), or 𝑔(2) = 14. The value of the expression 4𝑓(2) − 𝑔(2) can be found
ID: e77a76ce Answer

by substituting the corresponding values into the expression, which gives 4(9) − 14. This expression is
Correct Answer: A

equivalent to 36 − 14, or 22. Rationale

Choice A is incorrect. This is the value of 𝑓(2) − 𝑔(2), not 4𝑓(2) − 𝑔(2). Choice A is correct. A system of two linear equations in two variables, 𝑥 and 𝑦, has no solution if the graphs of

Choice B is incorrect and may result from calculating 4𝑓(2) as 4(2) + 7, rather than the xy-plane represented by equations in slope-intercept form, 𝑦 = 𝑚𝑥 + 𝑏, where 𝑚 and 𝑏 are constants, are
the lines represented by the equations in the xy-plane are distinct and parallel. The graphs of two lines in

4(2 + 7). parallel if their slopes, 𝑚, are the same and are distinct if their y-coordinates of the y-intercepts, 𝑏, are
different. In the equations 𝑦 = 6𝑥 + 3 and 𝑦 = 6𝑥 + 9, the values of 𝑚 are each 6, and the values of 𝑏 are 3 and 9,
Choice D is incorrect and may result from conceptual or calculation errors. respectively. Since the slopes of these lines are the same and the y-coordinates of the y-intercepts are
different, it follows that the system of linear equations in choice A has no solution.
Question Difficulty: Medium

of 10 that have the same y-coordinate of the y-intercept. Therefore, this system has a solution, (0, 10), rather
Choice B is incorrect. The two lines represented by these equations are a horizontal line and a line with a slope

than no solution.

coordinate of the y-intercept. Therefore, this system has a solution, (0, 14), rather than no solution.
Choice C is incorrect. The two lines represented by these equations have different slopes and the same y-

Therefore, this system has a solution, (3, 10), rather than no solution.
Choice D is incorrect. The two lines represented by these equations are a vertical line and a horizontal line.

Question Difficulty: Medium

about:srcdoc Page 39 of 126 about:srcdoc Page 40 of 126


10/30/23, 02:04 10/30/23, 02:04

limestone used in the garden is a product of its weight per volume, in lb/ft , and its volume, in ft . Therefore,
the weight of the limestone used in the garden can be represented by
3
, where x is the volume, in ft , of
! ! ! ! !!! ! !!! !! ! ! ! ! ! ! ! the limestone used. Similarly, the weight of the basalt used in the garden can be represented by , where y
3
is the volume, in ft , of the basalt used. It’s given that the total weight of the rocks used in the garden will be
! !!!!!! !!! ! !!! ! ! ! ! !! ! ! !!! ! !! ! ! !!!
1,000 pounds. Thus, the sum of the weights of the three rock types used is 1,000 pounds, which can be
represented by the equation . Subtracting 330 from both sides of this equation
!! ! ! ! !! ! !! ! ! ! ! ! !! ! ! ! !! ! ! ! !!! ! ! !!!
!! ! !! ! ! !! ! !! ! yields .

Choice A is incorrect. This equation uses the weight per volume of granite instead of limestone. Choice B is
ID: 28c2253f
incorrect. This equation uses the weight per volume of granite instead of basalt, and doesn’t take into account
the 330 pounds of granite that will be used in the garden. Choice D is incorrect. This equation doesn’t take into
Characteristics for Rock Types account the 330 pounds of granite that will be used in the garden.

Weight per volume Cost per Question Difficulty: Medium


Rock type 3
(lb/ft ) pound

Basalt 180 $0.18

Granite 165 $0.09

Limestone 120 $0.03

Sandstone 135 $0.22

A city is planning to build a rock retaining wall, a monument, and a garden in


a park. The table above shows four rock types that will be considered for
use in the project. Also shown for each rock type is its weight per volume, in
3
pounds per cubic foot (lb/ft ), and the cost per pound, in dollars. Only
basalt, granite, and limestone will be used in the garden. The rocks in the
garden will have a total weight of 1,000 pounds. If 330 pounds of granite is
used, which of the following equations could show the relationship between
3
the amounts, x and y, in ft , for each of the other rock types used?

A.

B.

C.

D.

ID: 28c2253f Answer


Correct Answer: C

Rationale

Choice C is correct. It’s given that the weight of the granite used in the garden is 330 pounds. The weight of the

3 3
limestone used in the garden is a product of its weight per volume, in lb/ft , and its volume, in ft . Therefore,
3
about:srcdoc Page 41 of 126 about:srcdoc Page 42 of 126
10/30/23, 02:04 10/30/23, 02:04

! ! ! ! !!! ! !!! !! ! ! ! ! ! ! ! ! ! ! ! !!! ! !!! !! ! ! ! ! ! ! !


! !!!!!! !!! ! !!! ! ! ! ! !! ! ! !!! ! !! ! ! !!! ! !!!!!! !!! ! !!! ! ! ! ! !! ! ! !!! ! !! ! ! !!!

!! ! ! ! !! ! !! ! ! ! ! ! !! ! ! ! !! ! ! ! !!! ! ! !!! !! ! ! ! !! ! !! ! ! ! ! ! !! ! ! ! !! ! ! ! !!! ! ! !!!


!! ! !! ! ! !! ! !! ! ! ! ! !! ! ! !! ! !!

ID: f81a0503 ID: ce6b52d8

In the xy-plane, line passes through the points and . Which equation defines line ? If , what is the value of ?

A.
ID: ce6b52d8 Answer
B.
Correct Answer: 60
C.
Rationale

The correct answer is 60. Subtracting 𝑡 from both sides of the given equation yields 2(3𝑡 − 10) = 40 + 3𝑡.
D.

Applying the distributive property to the left-hand side of this equation yields 6𝑡 − 20 = 40 + 3𝑡. Adding 20 to
both sides of this equation yields 6𝑡 = 60 + 3𝑡. Subtracting 3𝑡 from both sides of this equation yields 3𝑡 = 60.
Therefore, the value of 3𝑡 is 60.
ID: f81a0503 Answer
Correct Answer: D

Rationale Question Difficulty: Medium

Choice D is correct. An equation defining a line in the xy-plane can be written in the form 𝑦 = 𝑚𝑥 + 𝑏, where 𝑚
represents the slope and (0, 𝑏) represents the y-intercept of the line. It’s given that line 𝑘 passes through the
point (0, − 5); therefore, 𝑏 = − 5. The slope, 𝑚, of a line can be found using any two points on the line, (𝑥1 , 𝑦1 )
𝑦 − 𝑦1
and (𝑥2 , 𝑦2 ), and the slope formula 𝑚 = 2 . Substituting the points (0, − 5) and (1, − 1) for (𝑥1 , 𝑦1 ) and (𝑥2 , 𝑦2 )
𝑥2 − 𝑥1
(−1 − (−5))
, respectively, in the slope formula yields 𝑚 = , or 𝑚 = 4. Substituting 4 for 𝑚 and −5 for 𝑏 in the
(1 − 0)
equation 𝑦 = 𝑚𝑥 + 𝑏 yields 𝑦 = 4𝑥 − 5.

Choice A is incorrect and may result from conceptual or calculation errors.

Choice B is incorrect and may result from conceptual or calculation errors.

Choice C is incorrect and may result from conceptual or calculation errors.

Question Difficulty: Medium

about:srcdoc Page 43 of 126 about:srcdoc Page 44 of 126


10/30/23, 02:04 10/30/23, 02:04

! ! ! ! !!! ! !!! !! ! ! ! ! ! ! ! ! ! ! ! !!! ! !!! !! ! ! ! ! ! ! !


! !!!!!! !!! ! !!! ! ! ! ! !! ! ! !!! ! !! ! ! !!! ! !!!!!! !!! ! !!! ! ! ! ! !! ! ! !!! ! !! ! ! !!!

!! ! ! ! !! ! !! ! ! ! ! ! !! ! ! ! !! ! ! ! !!! ! ! !! ! ! ! !! ! !! ! ! ! ! ! !! ! ! ! !! ! ! ! !!! ! ! !!!


!! ! !! ! ! !! ! !! !

ID: 0cadb20e
ID: 265f2a53
The function is defined by , and , where is a constant. What is the value of ?
When line is graphed in the xy-plane, it has an x-intercept of and a y-intercept of . What is the
A. slope of line ?

B. A.

C. B.

D. C.

D.
ID: 0cadb20e Answer
Correct Answer: C
ID: 265f2a53 Answer
Rationale

𝑥 + 15
Correct Answer: C

Choice C is correct. It's given that 𝑓(𝑥) = and 𝑓(𝑎) = 10, where 𝑎 is a constant. Therefore, for the given
5
𝑎 + 15
Rationale
function 𝑓, when 𝑥 = 𝑎, 𝑓(𝑥) = 10. Substituting 𝑎 for 𝑥 and 10 for 𝑓(𝑥) in the given function 𝑓 yields 10 =
5 Choice C is correct. It's given that when line 𝑛 is graphed in the xy-plane, it has an x-intercept of (−4, 0) and a
.
Multiplying both sides of this equation by 5 yields 50 = 𝑎 + 15. Subtracting 15 from both sides of this equation 86
y-intercept of (0, ). The slope, 𝑚, of a line can be found using any two points on the line, (𝑥1 , 𝑦1 ) and (𝑥2 , 𝑦2 ),
yields 35 = 𝑎. Therefore, the value of 𝑎 is 35. 3
𝑦 − 𝑦1 86
and the slope formula 𝑚 = 2 . Substituting the points (−4, 0) and (0, ) for (𝑥1 , 𝑦1 ) and (𝑥2 , 𝑦2 ), respectively,
𝑥2 − 𝑥1 3
Choice A is incorrect. This is the value of 𝑎 if 𝑓(𝑎) = 4. 86
−0 43 43
in the slope formula yields 𝑚 = 3 , or 𝑚 = . Therefore, the slope of line 𝑛 is .
Choice B is incorrect. This is the value of 𝑎 if 𝑓(𝑎) = 5. 0 − (−4) 6 6

Choice D is incorrect. This is the value of 𝑎 if 𝑓(𝑎) = 16. Choice A is incorrect and may result from conceptual or calculation errors.

86
Choice B is incorrect. This is the slope of a line that has an x-intercept of ( , 0) and a y-intercept of (0, − 4).
3
Question Difficulty: Medium

Choice D is incorrect and may result from conceptual or calculation errors.

Question Difficulty: Medium

about:srcdoc Page 45 of 126 about:srcdoc Page 46 of 126


10/30/23, 02:04 10/30/23, 02:04

! ! ! ! !!! ! !!! !! ! ! ! ! ! ! ! ! ! ! ! !!! ! !!! !! ! ! ! ! ! ! !


! !!!!!! !!! ! !!! ! ! ! ! !! ! ! !!! ! !! ! ! !!! ! !!!!!! !!! ! !!! ! ! ! ! !! ! ! !!! ! !! ! ! !!!

!! ! ! ! !! ! !! ! ! ! ! ! ! ! !! ! ! !! ! !!! ! !! ! ! ! !! ! !! ! ! ! ! ! ! ! !! ! ! !! ! !!! !
!!! ! ! ! !! ! ! ! !!! ! ! !!! !!! ! ! ! !! ! ! ! !!! ! ! !!!
!! ! !! ! ! !! ! !! ! !! ! !! ! ! !! ! !! !

ID: 2875ba81 ID: 9f6f96ff

A wire with a length of inches is cut into two parts. One part has a length of inches, and the other part has a
length of inches. The value of is more than times the value of . What is the value of ?
The solution to the given system of equations is . What is the value of y?
A.
A.
B.
B.
C.
C.
D.
D.

ID: 9f6f96ff Answer


ID: 2875ba81 Answer Correct Answer: D
Correct Answer: A
Rationale

Choice D is correct. It's given that a wire with a length of 106 inches is cut into two parts. It's also given that
Rationale

one part has a length of 𝑥 inches and the other part has a length of 𝑦 inches. This can be represented by the
each side of the second equation in the given system by 3 yields (2𝑥 + 2𝑦)(3) = (10)(3), or 6𝑥 + 6𝑦 = 30. equation 𝑥 + 𝑦 = 106. It's also given that the value of 𝑥 is 6 more than 4 times the value of 𝑦. This can be
Choice A is correct. The given system of linear equations can be solved by the elimination method. Multiplying

Subtracting this equation from the first equation in the given system yields (6𝑥 + 7𝑦) − (6𝑥 + 6𝑦) = (28) − (30), represented by the equation 𝑥 = 4𝑦 + 6. Substituting 4𝑦 + 6 for 𝑥 in the equation 𝑥 + 𝑦 = 106 yields
which is equivalent to (6𝑥 − 6𝑥) + (7𝑦 − 6𝑦) = 28 − 30, or 𝑦 = − 2. 4𝑦 + 6 + 𝑦 = 106, or 5𝑦 + 6 = 106. Subtracting 6 from each side of this equation yields 5𝑦 = 100. Dividing each side
of this equation by 5 yields 𝑦 = 20. Substituting 20 for 𝑦 in the equation 𝑥 = 4𝑦 + 6 yields 𝑥 = 4(20) + 6, or 𝑥 = 86.
Choice B is incorrect. This is the value of 𝑥, not the value of 𝑦.
Choice A is incorrect. This value represents less than half of the total length of 106 inches; however, 𝑥
Choice C is incorrect and may result from conceptual or calculation errors. represents the length of the longer part of the wire, since it's given that the value of 𝑥 is 6 more than 4 times
the value of 𝑦.
Choice D is incorrect and may result from conceptual or calculation errors.
Choice B is incorrect. This value represents less than half of the total length of 106 inches; however, 𝑥
represents the length of the longer part of the wire, since it's given that the value of 𝑥 is 6 more than 4 times
Question Difficulty: Medium the value of 𝑦.

Choice C is incorrect. This represents a part that is 6 more than the length of the other part, rather than 6 more
than 4 times the length of the other part.

Question Difficulty: Medium

about:srcdoc Page 47 of 126 about:srcdoc Page 48 of 126


10/30/23, 02:04 10/30/23, 02:04

! ! ! ! !!! ! !!! !! ! ! ! ! ! ! ! ! ! ! ! !!! ! !!! !! ! ! ! ! ! ! !


! !!!!!! !!! ! !!! ! ! ! ! !! ! ! !!! ! !! ! ! !!! ! !!!!!! !!! ! !!! ! ! ! ! !! ! ! !!! ! !! ! ! !!!

!! ! ! ! !! ! !! ! ! ! ! ! !! ! ! ! !! ! ! ! !!! ! ! !! ! ! ! !! ! !! ! ! ! ! ! !! ! ! ! !! ! ! ! !!! ! ! !!!


! ! ! !! ! ! !! ! !!

ID: 7e1bff94
ID: ce314070
The table gives the number of hours, , of labor and a plumber’s total charge , in dollars, for two different
jobs.
If , what is the

value of ?

A.

There is a linear relationship between and . Which equation represents this relationship?
B.
A.

B. C.

C. D.

D.

ID: ce314070 Answer


ID: 7e1bff94 Answer Correct Answer: D
Correct Answer: C Rationale
Rationale

Choice C is correct. It's given that there is a linear relationship between a plumber's hours of labor, ℎ, and the Choice D is correct. Multiplying the given equation by 2 on each side yields . Applying the
plumber's total charge 𝑓(ℎ), in dollars. It follows that the relationship can be represented by an equation of the
form 𝑓(ℎ) = 𝑚ℎ + 𝑏, where 𝑚 is the rate of change of the function 𝑓 and 𝑏 is a constant. The rate of change of 𝑓 distributive property, this equation can be rewritten as , or .
can be calculated by dividing the difference in two values of 𝑓(ℎ) by the difference in the corresponding values
285 − 155
of ℎ. Based on the values given in the table, the rate of change of 𝑓 is , or 65. Substituting 65 for 𝑚 in
3−1
Choices A, B, and C are incorrect and may result from calculation errors in solving the given equation for and
the equation 𝑓(ℎ) = 𝑚ℎ + 𝑏 yields 𝑓(ℎ) = 65ℎ + 𝑏. The value of 𝑏 can be found by substituting any value of ℎ and
its corresponding value of 𝑓(ℎ) for ℎ and 𝑓(ℎ), respectively, in this equation. Substituting 1 for ℎ and 155 for 𝑓(ℎ)
then substituting that value of x in the expression .

yields 155 = 65(1) + 𝑏, or 155 = 65 + 𝑏. Subtracting 65 from both sides of this equation yields 90 = 𝑏. Substituting
90 for 𝑏 in the equation 𝑓(ℎ) = 65ℎ + 𝑏 yields 𝑓(ℎ) = 65ℎ + 90.
Question Difficulty: Medium

Choice A is incorrect and may result from conceptual or calculation errors.

Choice B is incorrect and may result from conceptual or calculation errors.

Choice D is incorrect and may result from conceptual or calculation errors.

Question Difficulty: Medium

about:srcdoc Page 49 of 126 about:srcdoc Page 50 of 126


10/30/23, 02:04 10/30/23, 02:04

! ! ! ! !!! ! !!! !! ! ! ! ! ! ! ! ! ! ! ! !!! ! !!! !! ! ! ! ! ! ! !


! !!!!!! !!! ! !!! ! ! ! ! !! ! ! !!! ! !! ! ! !!! ! !!!!!! !!! ! !!! ! ! ! ! !! ! ! !!! ! !! ! ! !!!

!! ! ! ! !! ! !! ! ! ! ! ! !! ! ! ! !! ! ! ! !!! ! ! !! ! ! ! !! ! !! ! ! ! ! ! !! ! ! ! !! ! ! ! !!! ! ! !!!


!! ! !! ! ! !! ! !! !

ID: 868fc236
ID: 431c3038

In an article about exercise, it is estimated that a 160-pound adult uses 200


calories for every 30 minutes of hiking and 150 calories for every
30 minutes of bicycling. An adult who weighs 160 pounds has completed
1 hour of bicycling. Based on the article, how many hours should the adult
The table above gives the typical amounts of energy per gram, expressed in hike to use a total of 1,900 calories from bicycling and hiking?
both food calories and kilojoules, of the three macronutrients in food. If x A. 9.5
food calories is equivalent to k kilojoules, of the following, which best
represents the relationship between x and k ? B. 8.75

A. C. 6

B. D. 4

C.
ID: 431c3038 Answer
D. Correct Answer: D

Rationale

ID: 868fc236 Answer


Choice D is correct. Since a 160-pound adult uses 200 calories for every 30 minutes of hiking, then the same
Correct Answer: B adult uses calories after hiking for h 30-minute periods. Similarly, the same adult uses calories
Rationale after bicycling for b 30-minute periods. Therefore, the equation represents the situation

where a 160-pound adult uses a total of 1,900 calories from hiking for h 30-minute periods and bicycling for b
Choice B is correct. The relationship between x food calories and k kilojoules can be modeled as a proportional
30-minute periods. It’s given that the adult completes 1 hour, or 2 30-minute periods, of bicycling. Substituting
relationship. Let and represent the values in the first two rows in the table: and
2 for b in the equation yields , or .

Subtracting 300 from both sides of this equation yields . Dividing both sides by 200 yields .
. The rate of change, or , is ; therefore, the equation that best represents the
Since h represents the number of 30-minute periods spent hiking and there are 2 30-minute periods in every
relationship between x and k is .
hour, it follows that the adult will need to hike for , or 4 hours to use a total of 1,900 calories from bicycling
and hiking.
Choice A is incorrect and may be the result of calculating the rate of change using . Choice C is

incorrect because the number of kilojoules is greater than the number of food calories. Choice D is incorrect Choice A is incorrect and may result from solving the equation . This represents 0 30-minute
and may be the result of an error when setting up the equation. periods bicycling instead of 2. Choice B is incorrect and may result from solving the equation
. This represents 1 30-minute period of bicycling instead of 2. Choice C is incorrect. This
Question Difficulty: Medium
may result from determining that the number of 30-minute periods the adult should hike is 8, but then

subtracting 2 from 8, rather than dividing 8 by 2, to find the number of hours the adult should hike.

about:srcdoc Page 51 of 126 about:srcdoc Page 52 of 126


10/30/23, 02:04 10/30/23, 02:04

! ! ! ! !!! ! !!! !! ! ! ! ! ! ! !
Question Difficulty: Medium
! !!!!!! !!! ! !!! ! ! ! ! !! ! ! !!! ! !! ! ! !!!

!! ! ! ! !! ! !! ! ! ! ! ! !! ! ! ! !! ! ! ! !!! ! !

ID: c01f4a95

For the linear function , is a constant and . What is the value of ?

ID: c01f4a95 Answer


Correct Answer: 39

Rationale

The correct answer is 39. It’s given that for the linear function 𝑗, 𝑚 is a constant and 𝑗(12) = 18. Substituting 12
for 𝑥 and 18 for 𝑗(𝑥) in the given equation yields 18 = 𝑚(12) + 144. Subtracting 144 from both sides of this
equation yields −126 = 𝑚(12). Dividing both sides of this equation by 12 yields −10.5 = 𝑚. Substituting −10.5 for
𝑚 in the given equation, 𝑗(𝑥) = 𝑚𝑥 + 144, yields 𝑗(𝑥) = − 10.5𝑥 + 144. Substituting 10 for 𝑥 in this equation yields
𝑗(10) = (−10.5)(10) + 144, or 𝑗(10) = 39. Therefore, the value of 𝑗(10) is 39.

Question Difficulty: Medium

about:srcdoc Page 53 of 126 about:srcdoc Page 54 of 126


10/30/23, 02:04 10/30/23, 02:04

! ! ! ! !!! ! !!! !! ! ! ! ! ! ! ! ! ! ! ! !!! ! !!! !! ! ! ! ! ! ! !


! !!!!!! !!! ! !!! ! ! ! ! !! ! ! !!! ! !! ! ! !!! ! !!!!!! !!! ! !!! ! ! ! ! !! ! ! !!! ! !! ! ! !!!

!! ! ! ! !! ! !! ! ! ! ! ! !! ! ! ! !!! ! ! ! ! !!!!! ! !! ! ! ! !! ! !! ! ! ! ! ! !! ! ! ! !! ! ! ! !!! ! ! !!!


!! !! ! ! !! ! !!! ! ! ! ! !! ! ! !! ! !!
! ! ! !! ! !! !

ID: 4f669597
ID: f2bbd43d

What value of p is the solution of


the equation above?
The point is a solution to the system of inequalities in the xy-plane. Which of the following could be the
value of ?

A. ID: 4f669597 Answer

B. Rationale

C. The correct answer is 1.2. One way to solve the equation is to first distribute the terms
outside the parentheses to the terms inside the parentheses: . Next, combine like terms
D.
on the left side of the equal sign: . Subtracting 10p from both sides yields . Finally,

ID: f2bbd43d Answer dividing both sides by gives , which is equivalent to . Note that 1.2 and 6/5 are examples of

Correct Answer: A ways to enter a correct answer.

Rationale
Question Difficulty: Medium

Choice A is correct. It’s given that the point (𝑥, 53) is a solution to the given system of inequalities in the xy-
plane. This means that the coordinates of the point, when substituted for the variables 𝑥 and 𝑦, make both of
the inequalities in the system true. Substituting 53 for 𝑦 in the inequality 𝑦 > 14 yields 53 > 14, which is true.
Substituting 53 for 𝑦 in the inequality 4𝑥 + 𝑦 < 18 yields 4𝑥 + 53 < 18. Subtracting 53 from both sides of this
inequality yields 4𝑥 < − 35. Dividing both sides of this inequality by 4 yields 𝑥 < − 8.75. Therefore, 𝑥 must be a
value less than −8.75. Of the given choices, only −9 is less than −8.75.

Choice B is incorrect. Substituting −5 for 𝑥 and 53 for 𝑦 in the inequality 4𝑥 + 𝑦 < 18 yields 4(−5) + 53 < 18, or
33 < 18, which is not true.

Choice C is incorrect. Substituting 5 for 𝑥 and 53 for 𝑦 in the inequality 4𝑥 + 𝑦 < 18 yields 4(5) + 53 < 18, or
73 < 18, which is not true.

Choice D is incorrect. Substituting 9 for 𝑥 and 53 for 𝑦 in the inequality 4𝑥 + 𝑦 < 18 yields 4(9) + 53 < 18, or
89 < 18, which is not true.

Question Difficulty: Medium

about:srcdoc Page 55 of 126 about:srcdoc Page 56 of 126


10/30/23, 02:04 10/30/23, 02:04

! ! ! ! !!! ! !!! !! ! ! ! ! ! ! ! ! ! ! ! !!! ! !!! !! ! ! ! ! ! ! !


! !!!!!! !!! ! !!! ! ! ! ! !! ! ! !!! ! !! ! ! !!! ! !!!!!! !!! ! !!! ! ! ! ! !! ! ! !!! ! !! ! ! !!!

!! ! ! ! !! ! !! ! ! ! ! ! !! ! ! ! !! ! ! ! !!! ! ! !! ! ! ! !! ! !! ! ! ! ! ! ! ! !! ! ! !! ! !!! !
!!! ! ! ! !! ! ! ! !!! ! ! !!!
!! ! !! ! ! !! ! !! !
ID: 56dc8045
ID: d909cd31

The function models the volume of liquid, in milliliters, in a container seconds after it begins draining from a
hole at the bottom. According to the model, what is the predicted volume, in milliliters, draining from the container
One of the two equations in a system of linear equations is given. The system has infinitely many solutions. Which
each second?
of the following could be the second equation in the system?
A.
A.
B.
B.
C.
C.
D.
D.

ID: 56dc8045 Answer


ID: d909cd31 Answer
Correct Answer: D
Correct Answer: C
Rationale
Rationale
Choice D is correct. It’s given that the function 𝑤 models the volume of liquid, in milliliters, in a container 𝑡
seconds after it begins draining from a hole at the bottom. The given function 𝑤(𝑡) = 300 − 4𝑡 can be rewritten
Choice C is correct. It's given that the system has infinitely many solutions. A system of two linear equations

as 𝑤(𝑡) = − 4𝑡 + 300. Thus, for each increase of 𝑡 by 1, the value of 𝑤(𝑡) decreases by 4(1), or 4. Therefore, the
equation by 5 yields − 3𝑥 + 5𝑦 = 13. Dividing both sides of choice C by 4 also yields − 3𝑥 + 5𝑦 = 13, so choice C
has infinitely many solutions when the two linear equations are equivalent. Dividing both sides of the given

predicted volume, in milliliters, draining from the container each second is 4 milliliters.
is equivalent to the given equation. Thus, choice C could be the second equation in the system.
Choice A is incorrect. This is the amount of liquid, in milliliters, in the container before the liquid begins
Choice A is incorrect. The system consisting of this equation and the given equation has one solution, not
draining.
infinitely many solutions.
Choice B is incorrect and may result from conceptual errors.
Choice B is incorrect. The system consisting of this equation and the given equation has one solution, not
Choice C is incorrect and may result from conceptual errors. infinitely many solutions.

Question Difficulty: Medium Choice D is incorrect. The system consisting of this equation and the given equation has no solution, not
infinitely many solutions.

Question Difficulty: Medium

about:srcdoc Page 57 of 126 about:srcdoc Page 58 of 126


10/30/23, 02:04 10/30/23, 02:04

! ! ! ! !!! ! !!! !! ! ! ! ! ! ! ! ! ! ! ! !!! ! !!! !! ! ! ! ! ! ! !


! !!!!!! !!! ! !!! ! ! ! ! !! ! ! !!! ! !! ! ! !!! ! !!!!!! !!! ! !!! ! ! ! ! !! ! ! !!! ! !! ! ! !!!

!! ! ! ! !! ! !! ! ! ! ! ! !! ! ! ! !! ! ! ! !!! ! ! !! ! ! ! !! ! !! ! ! ! ! ! !! ! ! ! !! ! ! ! !!! ! !

ID: c8fb6bcb ID: 23dedddd

In the linear function , and . Which equation defines ?


The given function represents the perimeter, in , of a rectangle with a length of and a
A.
fixed width. What is the width, in , of the rectangle?

A. B.

B. C.

C. D.

D.
ID: 23dedddd Answer
Correct Answer: D
ID: c8fb6bcb Answer
Rationale

Choice D is correct. Since 𝑓 is a linear function, it can be defined by an equation of the form 𝑓(𝑥) = 𝑎𝑥 + 𝑏,
Correct Answer: B

where 𝑎 and 𝑏 are constants. It's given that 𝑓(0) = 8. Substituting 0 for 𝑥 and 8 for 𝑓(𝑥) in the equation
Rationale

Choice B is correct. It's given that 𝑓(𝑥) = 2𝑥 + 244 represents the perimeter, in centimeters (cm), of a rectangle 𝑓(𝑥) = 𝑎𝑥 + 𝑏 yields 8 = 𝑎(0) + 𝑏, or 8 = 𝑏. Substituting 8 for 𝑏 in the equation 𝑓(𝑥) = 𝑎𝑥 + 𝑏 yields 𝑓(𝑥) = 𝑎𝑥 + 8.
with a length of 𝑥 cm and a fixed width. If 𝑤 represents a fixed width, in cm, then the perimeter, in cm, of a It's given that 𝑓(1) = 12. Substituting 1 for 𝑥 and 12 for 𝑓(𝑥) in the equation 𝑓(𝑥) = 𝑎𝑥 + 8 yields 12 = 𝑎(1) + 8, or
rectangle with a length of 𝑥 cm and a fixed width of 𝑤 cm can be given by the function 𝑓(𝑥) = 2𝑥 + 2𝑤. Therefore, 12 = 𝑎 + 8. Subtracting 8 from both sides of this equation yields 𝑎 = 4. Substituting 4 for 𝑎 in the equation
2𝑥 + 2𝑤 = 2𝑥 + 244. Subtracting 2𝑥 from both sides of this equation yields 2𝑤 = 244. Dividing both sides of this 𝑓(𝑥) = 𝑎𝑥 + 8 yields 𝑓(𝑥) = 4𝑥 + 8. Therefore, an equation that defines 𝑓 is 𝑓(𝑥) = 4𝑥 + 8.
equation by 2 yields 𝑤 = 122. Therefore, the width, in cm, of the rectangle is 122.
Choice A is incorrect and may result from conceptual or calculation errors.
Choice A is incorrect and may result from conceptual or calculation errors.
Choice B is incorrect and may result from conceptual or calculation errors.
Choice C is incorrect and may result from conceptual or calculation errors.
Choice C is incorrect and may result from conceptual or calculation errors.
Choice D is incorrect and may result from conceptual or calculation errors.
Question Difficulty: Medium
Question Difficulty: Medium

about:srcdoc Page 59 of 126 about:srcdoc Page 60 of 126


10/30/23, 02:04 10/30/23, 02:04

! ! ! ! !!! ! !!! !! ! ! ! ! ! ! ! ! ! ! ! !!! ! !!! !! ! ! ! ! ! ! !


! !!!!!! !!! ! !!! ! ! ! ! !! ! ! !!! ! !! ! ! !!! ! !!!!!! !!! ! !!! ! ! ! ! !! ! ! !!! ! !! ! ! !!!

!! ! ! ! !! ! !! ! ! ! ! ! !! ! ! ! !! ! ! ! !!! ! ! !! ! ! ! !! ! !! ! ! ! ! ! ! ! !! ! ! !! ! !!! !
!!! ! ! ! !! ! ! ! !!! ! ! !!!
!! ! !! ! ! !! ! !! !
ID: 3122fc7b

ID: 092ad67d
A linear model estimates the population of a city from to . The model estimates the population was
thousand in , thousand in , and thousand in . To the nearest whole number, what is the value
of ?

The solution to the given system of equations is . What is the value of ?


ID: 3122fc7b Answer
A.
Correct Answer: 257
B.
Rationale

The correct answer is 257. It’s given that a linear model estimates the population of a city from 1991 to 2015.
C.

for the model. It’s also given that the model estimates the population was 57 thousand in 1991, 224 thousand in
Since the population can be estimated using a linear model, it follows that there is a constant rate of change D.

2011, and 𝑥 thousand in 2015. The change in the population between 2011 and 1991 is 224 − 57, or 167, thousand.
The change in the number of years between 2011 and 1991 is 2011 − 1991, or 20, years. Dividing 167 by 20 gives ID: 092ad67d Answer
167/20, or 8.35, thousand per year. Thus, the change in population per year from 1991 to 2015 estimated by the
model is 8.35 thousand. The change in the number of years between 2015 and 2011 is 2015 − 2011, or 4, years.
Correct Answer: B

Rationale

population from 2011 to 2015 estimated by the model: (8.35)(4), or 33.4, thousand. Adding the change in
Multiplying the change in population per year by the change in number of years yields the increase in

population from 2011 to 2015 estimated by the model to the estimated population in 2011 yields the estimated
(𝑥 + 2𝑦) + (𝑥 − 2𝑦) = 6 + 4, or (𝑥 + 𝑥) + (2𝑦 − 2𝑦) = 10. Combining like terms in this equation yields 2𝑥 = 10.
Choice B is correct. Adding the first equation to the second equation in the given system yields

population in 2015. Thus, the estimated population in 2015 is 33.4 + 224, or 257.4, thousand. Therefore to the
Dividing both sides of this equation by 2 yields 𝑥 = 5. Thus, the value of 𝑥 is 5.
nearest whole number, the value of 𝑥 is 257.
Choice A is incorrect and may result from conceptual or calculation errors.
Question Difficulty: Medium
Choice C is incorrect and may result from conceptual or calculation errors.

Choice D is incorrect. This is the value of 2𝑥, not 𝑥.

Question Difficulty: Medium

about:srcdoc Page 61 of 126 about:srcdoc Page 62 of 126


10/30/23, 02:04 10/30/23, 02:04

! ! ! ! !!! ! !!! !! ! ! ! ! ! ! !
! !!!!!! !!! ! !!! ! ! ! ! !! ! ! !!! ! !! ! ! !!!

!! ! ! ! !! ! !! ! ! ! ! ! !! ! ! ! !! ! ! ! !!! ! ! !!!
!! ! !! ! ! !! ! !! !

ID: d62ad380
An artist paints and sells square tiles. The selling price P, in dollars, of a painted tile is a linear function of
the side length of the tile s, in inches, as shown in the table below.

Side length, s (inches) Price, P (dollars)

3 8.00

6 18.00

9 28.00

Which of the following could define the relationship between s and P ?

A.

B.

C.

D.

ID: d62ad380 Answer


Correct Answer: C

Rationale

Choice C is correct. The relationship between s and P can be modeled by a linear equation of the form P = ks +

a, where k and a are constants. The table shows that P increases by 10 when s increases by 3, so k = . To

solve for a, substitute one of the given pairs of values for s and P: when s = 3, P = 8, so , which

yields a = –2. The solution is therefore .

Choice A is incorrect. When s = 3, P = 8, but 3(3) + 10 = 19 ≠ 8. Choice B is incorrect. This may result from
using the first number given for P in the table as the constant term a in the linear equation P = ks + a, which is
true only when s = 0. Choice D is incorrect and may result from using the reciprocal of the slope of the line.

Question Difficulty: Medium

about:srcdoc Page 63 of 126 about:srcdoc Page 64 of 126


10/30/23, 02:04 10/30/23, 02:04

! ! ! ! !!! ! !!! !! ! ! ! ! ! ! ! ! ! ! ! !!! ! !!! !! ! ! ! ! ! ! !


! !!!!!! !!! ! !!! ! ! ! ! !! ! ! !!! ! !! ! ! !!! ! !!!!!! !!! ! !!! ! ! ! ! !! ! ! !!! ! !! ! ! !!!

!! ! ! ! !! ! !! ! ! ! ! ! !! ! ! ! !! ! ! ! !!! ! ! !!! !! ! ! ! !! ! !! ! ! ! ! ! !! ! ! ! !! ! ! ! !!! ! !


! ! ! !! ! ! !! ! !!

ID: 4fe4fd7c
ID: 620abf36

If , what is the value of ?


A company’s total cost , in dollars, to produce x shirts is given by the
A.
function above, where m is a constant and . The total cost to produce
B. 100 shirts is $800. What is the total cost, in dollars, to produce 1000 shirts?
(Disregard the $ sign when gridding your answer.)
C.

D. ID: 4fe4fd7c Answer

Rationale
ID: 620abf36 Answer
The correct answer is 3500. The given information includes a cost, $800, to produce 100 shirts. Substituting
Correct Answer: C
and into the given equation yields . Subtracting 500 from both sides of
Rationale
the equation yields . Dividing both sides of this equation by 100 yields . Substituting the
Choice C is correct. Subtracting 4(𝑥 + 4) from both sides of the given equation yields 1(𝑥 + 4) = 29, or 𝑥 + 4 = 29.
Therefore, the value of 𝑥 + 4 is 29.
value of m into the given equation yields . Substituting 1000 for x in this equation and solving
for gives the cost of 1000 shirts: , or 3500.

Choice A is incorrect and may result from conceptual or calculation errors.

Choice B is incorrect. This is the value of 𝑥, not 𝑥 + 4.


Question Difficulty: Medium

Choice D is incorrect and may result from conceptual or calculation errors.

Question Difficulty: Medium

about:srcdoc Page 65 of 126 about:srcdoc Page 66 of 126


10/30/23, 02:04 10/30/23, 02:04

! ! ! ! !!! ! !!! !! ! ! ! ! ! ! ! ! ! ! ! !!! ! !!! !! ! ! ! ! ! ! !


! !!!!!! !!! ! !!! ! ! ! ! !! ! ! !!! ! !! ! ! !!! ! !!!!!! !!! ! !!! ! ! ! ! !! ! ! !!! ! !! ! ! !!!

!! ! ! ! !! ! !! ! ! ! ! ! !! ! ! ! !! ! ! ! !!! ! ! !! ! ! ! !! ! !! ! ! ! ! ! !! ! ! ! !! ! ! ! !!! ! !

ID: 11e1ab81 ID: c22b5f25

In the xy-plane, the points and lie on the graph of

which of the following linear functions?

A.

B.

The graph above shows the distance traveled d, in feet, by a product on a


conveyor belt m minutes after the product is placed on the belt. Which of C.
the following equations correctly relates d and m ?

A. D.

B.
ID: c22b5f25 Answer
C. Correct Answer: C

Rationale
D.
Choice C is correct. A linear function can be written in the form , where m is the slope and b is

the y-coordinate of the y-intercept of the line. The slope of the graph can be found using the formula
ID: 11e1ab81 Answer
Correct Answer: A . Substituting the values of the given points into this formula yields or ,
Rationale
which simplifies to . Only choice C shows an equation with this slope.
Choice A is correct. The line passes through the origin. Therefore, this is a relationship of the form ,

where k is a constant representing the slope of the graph. To find the value of k, choose a point on the Choices A, B, and D are incorrect and may result from computation errors or misinterpreting the given
graph of the line other than the origin and substitute the values of m and d into the equation. For example, if information.
the point is chosen, then , and . Therefore, the equation of the line is .
Question Difficulty: Medium

Choice B is incorrect and may result from calculating the slope of the line as the change in time over the
change in distance traveled instead of the change in distance traveled over the change in time. Choices C and
D are incorrect because each of these equations represents a line with a d-intercept of 2. However, the graph
shows a line with a d-intercept of 0.

Question Difficulty: Medium

about:srcdoc Page 67 of 126 about:srcdoc Page 68 of 126


10/30/23, 02:04 10/30/23, 02:04

! ! ! ! !!! ! !!! !! ! ! ! ! ! ! ! ! ! ! ! !!! ! !!! !! ! ! ! ! ! ! !


! !!!!!! !!! ! !!! ! ! ! ! !! ! ! !!! ! !! ! ! !!! ! !!!!!! !!! ! !!! ! ! ! ! !! ! ! !!! ! !! ! ! !!!

!! ! ! ! !! ! !! ! ! ! ! ! !! ! ! ! !! ! ! ! !!! ! ! !! ! ! ! !! ! !! ! ! ! ! ! !! ! ! ! !! ! ! ! !!! ! ! !!!


!! ! !! ! ! !! ! !! !

ID: c651cc56
ID: 2e0290c3

Some values of the linear function f are shown in the table


above. What is the value of ?

A. 6

B. 7

C. 8

D. 9

ID: c651cc56 Answer


Correct Answer: B

Rationale

Choice B is correct. A linear function has a constant rate of change, and any two rows of the table shown can The graph shows the relationship between the number of shares of stock from Company A, , and the number of
be used to calculate this rate. From the first row to the second, the value of x is increased by 2 and the value of shares of stock from Company B, , that Simone can purchase. Which equation could represent this relationship?
is increased by . So the values of increase by 3 for every increase by 1 in the value of x.
A.
Since , it follows that . Therefore, .
B.

Choice A is incorrect. This is the third x-value in the table, not . Choices C and D are incorrect and may
C.
result from errors when calculating the function’s rate of change.
D.
Question Difficulty: Medium

ID: 2e0290c3 Answer


Correct Answer: B

Rationale

Choice B is correct. The graph shown is a line passing through the points (0, 40) and (60, 0). Since the
relationship between 𝑥 and 𝑦 is linear, if two points on the graph make a linear equation true, then the
equation represents the relationship. Substituting 0 for 𝑥 and 40 for 𝑦 in the equation in choice B, 8𝑥 + 12𝑦 = 480,
yields 8(0) + 12(40) = 480, or 480 = 480, which is true. Substituting 60 for 𝑥 and 0 for 𝑦 in the equation
8𝑥 + 12𝑦 = 480 yields 8(60) + 12(0) = 480, or 480 = 480, which is true. Therefore, the equation 8𝑥 + 12𝑦 = 480

about:srcdoc Page 69 of 126 about:srcdoc Page 70 of 126


10/30/23, 02:04 10/30/23, 02:04

represents the relationship between 𝑥 and 𝑦.

Choice A is incorrect. The point (0, 40) is not on the graph of this equation, since 40 = 8(0) + 12, or 40 = 12, is not ! ! ! ! !!! ! !!! !! ! ! ! ! ! ! !
true.
! !!!!!! !!! ! !!! ! ! ! ! !! ! ! !!! ! !! ! ! !!!
Choice C is incorrect. The point (0, 40) is not on the graph of this equation, since 40 = 12(0) + 8, or 40 = 8, is not
true. !! ! ! ! !! ! !! ! ! ! ! ! ! ! !! ! ! !! ! !!! !

Choice D is incorrect. The point (0, 40) is not on the graph of this equation, since 12(0) + 8(40) = 480, or 320 = 480,
!!! ! ! ! !! ! ! ! !!! ! ! !!!
!! ! !! ! ! !! ! !! !
is not true.

Question Difficulty: Medium ID: c5082ce3

The score on a trivia game is obtained by subtracting the number of


incorrect answers from twice the number of correct answers. If a player
answered 40 questions and obtained a score of 50, how many questions
did the player answer correctly?

ID: c5082ce3 Answer

Rationale

The correct answer is 30. Let x represent the number of correct answers from the player and y represent the
number of incorrect answers from the player. Since the player answered 40 questions in total, the equation
represents this situation. Also, since the score is found by subtracting the number of incorrect
answers from twice the number of correct answers and the player received a score of 50, the equation
represents this situation. Adding the equations in the system of two equations together yields
. This can be rewritten as . Finally, solving for x by dividing both sides of the

equation by 3 yields .

Question Difficulty: Medium

about:srcdoc Page 71 of 126 about:srcdoc Page 72 of 126


10/30/23, 02:04 10/30/23, 02:04

! ! ! ! !!! ! !!! !! ! ! ! ! ! ! ! ! ! ! ! !!! ! !!! !! ! ! ! ! ! ! !


! !!!!!! !!! ! !!! ! ! ! ! !! ! ! !!! ! !! ! ! !!! ! !!!!!! !!! ! !!! ! ! ! ! !! ! ! !!! ! !! ! ! !!!

!! ! ! ! !! ! !! ! ! ! ! ! !! ! ! ! !!! ! ! ! ! !!!!! ! !! ! ! ! !! ! !! ! ! ! ! ! !! ! ! ! !! ! ! ! !!! ! ! !!!


!! !! ! ! !! ! !!! ! ! ! ! !! ! ! !! ! !!
! ! ! !! ! !! !

ID: f09097b1
ID: c17d9ba9
An agricultural scientist studying the growth of corn plants recorded the
A number is at most less than times the value of . If the value of is , what is the greatest possible value height of a corn plant at the beginning of a study and the height of the plant
of ? each day for the next 12 days. The scientist found that the height of the
plant increased by an average of 1.20 centimeters per day for the 12 days. If
ID: c17d9ba9 Answer the height of the plant on the last day of the study was 36.8 centimeters,
what was the height, in centimeters, of the corn plant at the beginning of
Correct Answer: -2
the study?
Rationale

The correct answer is −2. It's given that a number 𝑥 is at most 17 less than 5 times the value of 𝑦, or 𝑥 ≤ 5𝑦 − 17. ID: f09097b1 Answer
Substituting 3 for 𝑦 in this inequality yields 𝑥 ≤ 5(3) − 17, or 𝑥 ≤ − 2. Thus, if the value of 𝑦 is 3, the greatest
possible value of 𝑥 is −2.
Rationale

The correct answer is 22.4. If the height of the plant increased by an average of 1.20 centimeters per day for 12
Question Difficulty: Medium
days, then its total growth over the 12 days was centimeters. The plant was 36.8 centimeters

tall after 12 days, so at the beginning of the study its height was centimeters. Note that 22.4
and 112/5 are examples of ways to enter a correct answer.

Alternate approach: The equation can be used to represent this situation, where h is the

height of the plant, in centimeters, at the beginning of the study. Solving this equation for h yields 22.4
centimeters.

Question Difficulty: Medium

about:srcdoc Page 73 of 126 about:srcdoc Page 74 of 126


10/30/23, 02:04 10/30/23, 02:04

! ! ! ! !!! ! !!! !! ! ! ! ! ! ! ! ! ! ! ! !!! ! !!! !! ! ! ! ! ! ! !


! !!!!!! !!! ! !!! ! ! ! ! !! ! ! !!! ! !! ! ! !!! ! !!!!!! !!! ! !!! ! ! ! ! !! ! ! !!! ! !! ! ! !!!

!! ! ! ! !! ! !! ! ! ! ! ! !! ! ! ! !!! ! ! ! ! !!!!! ! !! ! ! ! !! ! !! ! ! ! ! ! !! ! ! ! !! ! ! ! !!! ! ! !!!


!! !! ! ! !! ! !!! ! ! ! ! !! ! ! !! ! !!
! ! ! !! ! !! !

ID: 70e29454
ID: b31c3117

In the equation above, a and b are constants. If the equation has infinitely
The Karvonen formula above shows the relationship between Alice’s target many solutions, what are the values of a and b ?
heart rate H, in beats per minute (bpm), and the intensity level p of different
activities. When , Alice has a resting heart rate. When , Alice has A. and

her maximum heart rate. It is recommended that p be between 0.5 and 0.85 B. and
for Alice when she trains. Which of the following inequalities describes
Alice’s target training heart rate? C. and
A.
D. and
B.

C. ID: 70e29454 Answer


Correct Answer: B
D.
Rationale

Choice B is correct. Distributing the a on the left-hand side of the equation gives 3a – b – ax = –1 – 2x.
ID: b31c3117 Answer
Rearranging the terms in each side of the equation yields –ax + 3a – b = –2x –1. Since the equation has
Correct Answer: A infinitely many solutions, it follows that the coefficients of x and the free terms on both sides must be equal.
Rationale That is, –a = –2, or a = 2, and 3a – b = –1. Substituting 2 for a in the equation 3a – b = –1 gives 3(2) – b = –1, so b
= 7.
Choice A is correct. When Alice trains, it’s recommended that p be between 0.5 and 0.85. Therefore, her target
training heart rate is represented by the values of H corresponding to . When , Choice A is incorrect and may be the result of a conceptual error when finding the value of b. Choices C and D
are incorrect and may result from making a sign error when simplifying.
, or . When , , or . Therefore, the inequality that
describes Alice’s target training heart rate is . Question Difficulty: Medium

Choice B is incorrect. This inequality describes Alice’s target heart rate for . Choice C is
incorrect. This inequality describes her target heart rate for . Choice D is incorrect. This inequality
describes her target heart rate for .

Question Difficulty: Medium

about:srcdoc Page 75 of 126 about:srcdoc Page 76 of 126


10/30/23, 02:04 10/30/23, 02:04

! ! ! ! !!! ! !!! !! ! ! ! ! ! ! ! ! ! ! ! !!! ! !!! !! ! ! ! ! ! ! !


! !!!!!! !!! ! !!! ! ! ! ! !! ! ! !!! ! !! ! ! !!! ! !!!!!! !!! ! !!! ! ! ! ! !! ! ! !!! ! !! ! ! !!!

!! ! ! ! !! ! !! ! ! ! ! ! !! ! ! ! !! ! ! ! !!! ! ! !! ! ! ! !! ! !! ! ! ! ! ! !! ! ! ! !!! ! ! ! ! !!!!! !


!! !! ! ! !! ! !!! !
! ! ! !! ! !! !
ID: 271f7e3f
ID: 80da233d

A certain elephant weighs 200 pounds at birth and gains more than 2 but
For the function f defined above, what is the less than 3 pounds per day during its first year. Which of the following
value of ? inequalities represents all possible weights w, in pounds, for the elephant
365 days after birth?
A.
A.
B.
B.

C. C.

D.
D.

ID: 80da233d Answer


ID: 271f7e3f Answer Correct Answer: D
Correct Answer: B
Rationale
Rationale
Choice D is correct. It’s given that the elephant weighs 200 pounds at birth and gains more than 2 pounds but
Choice B is correct. The value of can be calculated by finding the values of and . The value less than 3 pounds per day during its first year. The inequality represents this

situation, where d is the number of days after birth. Substituting 365 for d in the inequality gives
of can be found by substituting 9 for x in the given function: . This equation can be , or .

rewritten as , or 4. Then, the value of can be found by substituting 1 for x in the given function: Choice A is incorrect and may result from solving the inequality . Choice B is incorrect
and may result from solving the inequality for a weight range of more than 1 pound but less than 2 pounds:
. This equation can be rewritten as , or 2. Therefore, , which is . Choice C is incorrect and may result from calculating the possible weight

equivalent to 2. gained by the elephant during the first year without adding the 200 pounds the elephant weighed at birth.

Choices A, C, and D are incorrect and may result from incorrectly substituting values of x in the given function Question Difficulty: Medium
or making computational errors.

Question Difficulty: Medium

about:srcdoc Page 77 of 126 about:srcdoc Page 78 of 126


10/30/23, 02:04 10/30/23, 02:04

Therefore, an equation of the graph shown is 𝑦 = − 𝑥 − 8.

! ! ! ! !!! ! !!! !! ! ! ! ! ! ! ! Choice A is incorrect. This is an equation of a line with a slope of −2, not −1.

! !!!!!! !!! ! !!! ! ! ! ! !! ! ! !!! ! !! ! ! !!! Choice B is incorrect. This is an equation of a line with a slope of 1, not −1.

Choice D is incorrect. This is an equation of a line with a slope of 2, not −1.


!! ! ! ! !! ! !! ! ! ! ! ! !! ! ! ! !! ! ! ! !!! ! ! !!!
!! ! !! ! ! !! ! !! !
Question Difficulty: Medium

ID: c307283c

What is an equation of the graph shown?

A.

B.

C.

D.

ID: c307283c Answer


Correct Answer: C

Rationale

Choice C is correct. An equation of a line can be written in the form 𝑦 = 𝑚𝑥 + 𝑏, where 𝑚 is the slope of the line
and (0, 𝑏) is the y-intercept of the line. The line shown passes through the point (0, − 8), so 𝑏 = − 8. The line
shown also passes through the point (−8, 0). The slope, 𝑚, of a line passing through two points (𝑥1 , 𝑦1 ) and
𝑦 − 𝑦1
(𝑥2 , 𝑦2 ) can be calculated using the equation 𝑚 = 2 . For the points (0, − 8) and (−8, 0), this gives
𝑥2 − 𝑥1
(−8) − 0
𝑚= , or 𝑚 = − 1. Substituting −8 for 𝑏 and −1 for 𝑚 in 𝑦 = 𝑚𝑥 + 𝑏 yields 𝑦 = (−1)𝑥 + (−8), or 𝑦 = − 𝑥 − 8.
0 − (−8)

about:srcdoc Page 79 of 126 about:srcdoc Page 80 of 126


10/30/23, 02:04 10/30/23, 02:04

! ! ! ! !!! ! !!! !! ! ! ! ! ! ! ! ! ! ! ! !!! ! !!! !! ! ! ! ! ! ! !


! !!!!!! !!! ! !!! ! ! ! ! !! ! ! !!! ! !! ! ! !!! ! !!!!!! !!! ! !!! ! ! ! ! !! ! ! !!! ! !! ! ! !!!

!! ! ! ! !! ! !! ! ! ! ! ! !! ! ! ! !!! ! ! ! ! !!!!! ! !! ! ! ! !! ! !! ! ! ! ! ! !! ! ! ! !! ! ! ! !!! ! !


!! !! ! ! !! ! !!! !
! ! ! !! ! !! !
ID: dae126d7
ID: bf5f80c6
The boiling point of water at sea level is 212 degrees Fahrenheit . For
every 550 feet above sea level, the boiling point of water is lowered by
Which point ( , ) is a solution to the given inequality in the -plane? about . Which of the following equations can be used to find the boiling

A. ( , ) point B of water, in , x feet above sea level?

B. ( , ) A.

C. ( , )
B.
D. ( , )

C.
ID: bf5f80c6 Answer
Correct Answer: A D.

Rationale

Choice D is correct. For a point (𝑥, 𝑦) to be a solution to the given inequality in the xy-plane, the value of the ID: dae126d7 Answer
point’s y-coordinate must be less than the value of −4𝑥 + 4, where 𝑥 is the value of the x-coordinate of the
point. This is true of the point (−4, 0) because 0 < − 4(−4) + 4, or 0 < 20. Therefore, the point (−4, 0) is a solution
Correct Answer: D

Rationale
to the given inequality.
Choice D is correct. It’s given that the boiling point of water at sea level is 212°F and that for every 550 feet
above sea level, the boiling point of water is lowered by about 1°F. Therefore, the change in the boiling point of

point’s y-coordinate is greater than the value of −4𝑥 + 4 for the point’s x-coordinate.
Choices A, B, and C are incorrect. None of these points are a solution to the given inequality because each
water x feet above sea level is represented by the expression . Adding this expression to the boiling
point of water at sea level gives the equation for the boiling point B of water, in °F, x feet above sea level:
Question Difficulty: Medium
, or .

Choices A and B are incorrect and may result from using the boiling point of water at sea level as the rate of
change and the rate of change as the initial boiling point of water at sea level. Choice C is incorrect and may
result from representing the change in the boiling point of water as an increase rather than a decrease.

Question Difficulty: Medium

about:srcdoc Page 81 of 126 about:srcdoc Page 82 of 126


10/30/23, 02:04 10/30/23, 02:04

! ! ! ! !!! ! !!! !! ! ! ! ! ! ! ! ! ! ! ! !!! ! !!! !! ! ! ! ! ! ! !


! !!!!!! !!! ! !!! ! ! ! ! !! ! ! !!! ! !! ! ! !!! ! !!!!!! !!! ! !!! ! ! ! ! !! ! ! !!! ! !! ! ! !!!

!! ! ! ! !! ! !! ! ! ! ! ! !! ! ! ! !! ! ! ! !!! ! ! !! ! ! ! !! ! !! ! ! ! ! ! ! ! !! ! ! !! ! !!! !
!!! ! ! ! !! ! ! ! !!! ! ! !!!
!! ! !! ! ! !! ! !! !
ID: a775af14

ID: 19fdf387
In the xy-plane, the graph of the linear function contains the points and . Which equation defines
, where ?
In the xy-plane, the graph of intersects the graph of at
A.
the point . What is the value of a ?
B.
A. 3
C.
B. 6
D.
C. 9

D. 12
ID: a775af14 Answer
Correct Answer: C
ID: 19fdf387 Answer
Rationale

Choice C is correct. In the xy-plane, the graph of a linear function can be written in the form 𝑓(𝑥) = 𝑚𝑥 + 𝑏,
Correct Answer: C

where 𝑚 represents the slope and (0, 𝑏) represents the y-intercept of the graph of 𝑦 = 𝑓(𝑥). It’s given that the
Rationale

graph of the linear function 𝑓, where 𝑦 = 𝑓(𝑥), in the xy-plane contains the point (0, 2). Thus, 𝑏 = 2. The slope of
the graph of a line containing any two points (𝑥1 , 𝑦1 ) and (𝑥2 , 𝑦2 ) can be found using the slope formula,
Choice C is correct. Since the graph of intersects the graph of at the point , the

𝑦 − 𝑦1
𝑚= 2 . Since it’s given that the graph of the linear function 𝑓 contains the points (0, 2) and (8, 34), it
𝑥2 − 𝑥1
ordered pair is the solution to the system of linear equations consisting of and , and

34 − 2
follows that the slope of the graph of the line containing these points is 𝑚 = , or 𝑚 = 4. Substituting 4 for
8−0
the value of a is the value of x in the solution of this system. Since both and are equal to y, it

( ) ( )
𝑚 and 2 for 𝑏 in 𝑓 𝑥 = 𝑚𝑥 + 𝑏 yields 𝑓 𝑥 = 4𝑥 + 2.
follows that . Subtracting x from and adding 6 to both sides of the equation yields .

Choice A is incorrect. This function represents a graph with a slope of 2 and a y-intercept of (0, 42).
Therefore, the value of a is 9.

Choice B is incorrect. This function represents a graph with a slope of 32 and a y-intercept of (0, 36).
Choices A and B are incorrect and may result from a calculation or conceptual error in solving the system of
equations consisting of and . Choice D is incorrect. This is the value of b, not a.

Choice D is incorrect. This function represents a graph with a slope of 8 and a y-intercept of (0, 2).
Question Difficulty: Medium
Question Difficulty: Medium

about:srcdoc Page 83 of 126 about:srcdoc Page 84 of 126


10/30/23, 02:04 10/30/23, 02:04

! ! ! ! !!! ! !!! !! ! ! ! ! ! ! ! ! ! ! ! !!! ! !!! !! ! ! ! ! ! ! !


! !!!!!! !!! ! !!! ! ! ! ! !! ! ! !!! ! !! ! ! !!! ! !!!!!! !!! ! !!! ! ! ! ! !! ! ! !!! ! !! ! ! !!!

!! ! ! ! !! ! !! ! ! ! ! ! !! ! ! ! !! ! ! ! !!! ! ! !! ! ! ! !! ! !! ! ! ! ! ! ! ! !! ! ! !! ! !!! !
!!! ! ! ! !! ! ! ! !!! ! ! !!!
!! ! !! ! ! !! ! !! !
ID: 3e9eaffc
ID: ed92fb68
Caleb used juice to make popsicles. The function approximates the volume, in fluid ounces, of
juice Caleb had remaining after making popsicles. Which statement is the best interpretation of the y-intercept
of the graph of in the xy-plane in this context?

A. Caleb used approximately fluid ounces of juice for each popsicle. If the system of equations above has solution ,

B. Caleb had approximately fluid ounces of juice when he began to make the popsicles. what is the value of ?

C. Caleb had approximately fluid ounces of juice when he began to make the popsicles. A. 0

D. Caleb used approximately fluid ounces of juice for each popsicle. B. 9

C. 18
ID: 3e9eaffc Answer
D. 38
Correct Answer: C

Rationale
ID: ed92fb68 Answer
Choice C is correct. An equation that defines a linear function 𝑓 can be written in the form 𝑓(𝑥) = 𝑚𝑥 + 𝑏, where
𝑚 represents the slope and 𝑏 represents the y-intercept, (0, 𝑏), of the line of 𝑦 = 𝑓(𝑥) in the xy-plane. The
Correct Answer: C

function 𝑓(𝑥) = − 5𝑥 + 30 is linear. Therefore, the graph of the given function 𝑦 = 𝑓(𝑥) in the xy-plane has a y-
Rationale

intercept of (0, 30). It’s given that 𝑓(𝑥) gives the approximate volume, in #uid ounces, of juice Caleb had
remaining after making 𝑥 popsicles. It follows that the y-intercept of (0, 30) means that Caleb had
Choice C is correct. Adding the given equations yields 9x + 9y = 162. Dividing each side of the equation 9x + 9y

approximately 30 #uid ounces of juice remaining after making 0 popsicles. In other words, Caleb had
= 162 by 9 gives x + y = 18.

approximately 30 #uid ounces of juice when he began to make the popsicles. Choice A is incorrect and may result from incorrectly adding the equations. Choice B is incorrect and may

Choice A is incorrect. This is an interpretation of the slope, rather than the y-intercept, of the graph of 𝑦 = 𝑓(𝑥)
result from conceptual or computational errors. Choice D is incorrect. This value is equivalent to y – x.

in the xy-plane. Question Difficulty: Medium

Choice B is incorrect and may result from conceptual errors.

Choice D is incorrect and may result from conceptual errors.

Question Difficulty: Medium

about:srcdoc Page 85 of 126 about:srcdoc Page 86 of 126


10/30/23, 02:04 10/30/23, 02:04

! ! ! ! !!! ! !!! !! ! ! ! ! ! ! ! ! ! ! ! !!! ! !!! !! ! ! ! ! ! ! !


! !!!!!! !!! ! !!! ! ! ! ! !! ! ! !!! ! !! ! ! !!! ! !!!!!! !!! ! !!! ! ! ! ! !! ! ! !!! ! !! ! ! !!!

!! ! ! ! !! ! !! ! ! ! ! ! ! ! !! ! ! !! ! !!! ! !! ! ! ! !! ! !! ! ! ! ! ! !! ! ! ! !! ! ! ! !!! ! ! !!!


!!! ! ! ! !! ! ! ! !!! ! ! !!! !! ! !! ! ! !! ! !! !
!! ! !! ! ! !! ! !! !

ID: 9c7741c6
ID: 8a87c2c8
On a 210-mile trip, Cameron drove at an average speed of 60 miles per
hour for the first x hours. He then completed the trip, driving at an average
speed of 50 miles per hour for the remaining y hours. If , what is the
The solution to the given system of equations is . What is the value of ?
value of y ?
A.

B. ID: 9c7741c6 Answer

C. Rationale

D. The correct answer is 3. It’s given that Cameron drove 60 miles per hour for x hours; therefore, the distance
driven at this speed can be represented by . He then drove 50 miles per hour for y hours; therefore, the
distance driven at this speed can be represented by . Since Cameron drove 210 total miles, the equation
ID: 8a87c2c8 Answer
represents this situation. If , substitution yields , or .
Correct Answer: C
Subtracting 60 from both sides of this equation yields . Dividing both sides of this equation by 50
Rationale
yields .

yields (𝑥 + 3) + (𝑥 − 3) = (−2𝑦 + 5) + (2𝑦 + 7). Adding like terms in this equation yields 2𝑥 = 12. Thus, the value
Choice C is correct. Adding the second equation in the given system to the first equation in the given system

of 2𝑥 is 12.
Question Difficulty: Medium

Choice A is incorrect. This is the value of 𝑦, not 2𝑥.

Choice B is incorrect. This is the value of 𝑥, not 2𝑥.

Choice D is incorrect and may result from conceptual or calculation errors.

Question Difficulty: Medium

about:srcdoc Page 87 of 126 about:srcdoc Page 88 of 126


10/30/23, 02:04 10/30/23, 02:04

! ! ! ! !!! ! !!! !! ! ! ! ! ! ! !
! !!!!!! !!! ! !!! ! ! ! ! !! ! ! !!! ! !! ! ! !!!

!! ! ! ! !! ! !! ! ! ! ! ! ! ! !! ! ! !! ! !!! !
!!! ! ! ! !! ! ! ! !!! ! ! !!!
!! ! !! ! ! !! ! !! !

ID: 74c03c21

A bus traveled on the highway and on local roads to complete a trip of . The trip took . The bus
traveled at an average speed of on the highway and an average speed of on
local roads. If is the time, in hours, the bus traveled on the highway and is the time, in hours, it traveled on
local roads, which system of equations represents this situation?

A.

B.

C.

D.

ID: 74c03c21 Answer


Correct Answer: B

Rationale

Choice B is correct. If the bus traveled at an average speed of 55 miles per hour (mph) on the highway for 𝑥
hours, then the bus traveled 55𝑥 miles on the highway. If the bus traveled at an average speed of 25 mph on
local roads for 𝑦 hours, then the bus traveled 25𝑦 miles on local roads. It's given that the trip was 160 miles. This
can be represented by the equation 55𝑥 + 25𝑦 = 160. It's also given that the trip took 4 hours. This can be
represented by the equation 𝑥 + 𝑦 = 4. Therefore, the system consisting of the equations 55𝑥 + 25𝑦 = 160 and
𝑥 + 𝑦 = 4 represents this situation.

Choice A is incorrect. This system of equations represents a situation where the trip was 4 miles and took 160
hours.

Choice C is incorrect. This system of equations represents a situation where the trip was 4 miles and took 160
hours, and the bus traveled at an average speed of 25 mph on the highway and 55 mph on local roads.

speed of 25 mph on the highway and 55 mph on local roads.


Choice D is incorrect. This system of equations represents a situation where the bus traveled at an average

Question Difficulty: Medium

about:srcdoc Page 89 of 126 about:srcdoc Page 90 of 126


10/30/23, 02:04 10/30/23, 02:04

Choice B is incorrect. This is an interpretation of the y-intercept of the graph rather than the slope of the

! ! ! ! !!! ! !!! !! ! ! ! ! ! ! ! graph.

Choice C is incorrect. The slope of the graph is the cost per game, not the cost of the video game system.
! !!!!!! !!! ! !!! ! ! ! ! !! ! ! !!! ! !! ! ! !!!
Choice D is incorrect. Each game costs $ 25, not $ 100.
!! ! ! ! !! ! !! ! ! ! ! ! !! ! ! ! !! ! ! ! !!! ! !
Question Difficulty: Medium

ID: 5cf1bbc9

The graph of the function , where , gives the total cost , in dollars, for a certain video game system
and games. What is the best interpretation of the slope of the graph in this context?

A. Each game costs .

B. The video game system costs .

C. The video game system costs .

D. Each game costs .

ID: 5cf1bbc9 Answer


Correct Answer: A

Rationale

Choice A is correct. The given graph is a line, and the slope of a line is defined as the change in the value of 𝑦
for each increase in the value of 𝑥 by 1. It’s given that 𝑦 represents the total cost, in dollars, and that 𝑥
represents the number of games. Therefore, the change in the value of 𝑦 for each increase in the value of 𝑥 by 1
represents the change in total cost, in dollars, for each increase in the number of games by 1. In other words,
the slope represents the cost, in dollars, per game. The graph shows that when the value of 𝑥 increases from 0
to 1, the value of 𝑦 increases from 100 to 125. It follows that the slope is 25, or the cost per game is $ 25. Thus,
the best interpretation of the slope of the graph is that each game costs $ 25.

about:srcdoc Page 91 of 126 about:srcdoc Page 92 of 126


10/30/23, 02:04 10/30/23, 02:04

! ! ! ! !!! ! !!! !! ! ! ! ! ! ! ! ! ! ! ! !!! ! !!! !! ! ! ! ! ! ! !


! !!!!!! !!! ! !!! ! ! ! ! !! ! ! !!! ! !! ! ! !!! ! !!!!!! !!! ! !!! ! ! ! ! !! ! ! !!! ! !! ! ! !!!

!! ! ! ! !! ! !! ! ! ! ! ! ! ! !! ! ! !! ! !!! ! !! ! ! ! !! ! !! ! ! ! ! ! !! ! ! ! !! ! ! ! !!! ! !
!!! ! ! ! !! ! ! ! !!! ! ! !!!
!! ! !! ! ! !! ! !! !
ID: f7e39fe9

ID: f5929f7a

The table shows four values of and their corresponding values of . There is a linear relationship between
The solution to the given system of equations is . What is the value of ?
and that is defined by the equation , where is a constant. What is the value of ?
A.

B. ID: f7e39fe9 Answer


Correct Answer: 11
C.
Rationale

The correct answer is 11. It's given that 𝑓(𝑥) is defined by the equation 𝑓(𝑥) = 𝑚𝑥 − 28, where 𝑚 is a constant.
D.

It's also given in the table that when 𝑥 = 10, 𝑓(𝑥) = 82. Substituting 10 for 𝑥 and 82 for 𝑓(𝑥) in the equation
ID: f5929f7a Answer 𝑓(𝑥) = 𝑚𝑥 − 28 yields, 82 = 𝑚(10) − 28. Adding 28 to both sides of this equation yields 110 = 10𝑚. Dividing both
Correct Answer: C sides of this equation by 10 yields 11 = 𝑚. Therefore, the value of 𝑚 is 11.

Rationale Question Difficulty: Medium


1 1
Choice C is correct. It's given by the first equation in the system that 𝑦 = − 𝑥. Substituting − 𝑥 for 𝑦 in the
9 9
1 1 2
second equation in the system yields − 𝑥 = 𝑥. Multiplying the left-hand side of this equation by and the
9 2 2
9 2 9 2 11
right-hand side by yields − 𝑥 = 𝑥. Adding 𝑥 to both sides of this equation yields 0 = 𝑥. Multiplying
9 18 18 18 18
18
both sides of this equation by yields 𝑥 = 0.
11

Choice A is incorrect and may result from conceptual or calculation errors.

Choice B is incorrect and may result from conceptual or calculation errors.

Choice D is incorrect and may result from conceptual or calculation errors.

Question Difficulty: Medium

about:srcdoc Page 93 of 126 about:srcdoc Page 94 of 126


10/30/23, 02:04 10/30/23, 02:04

! ! ! ! !!! ! !!! !! ! ! ! ! ! ! ! ! ! ! ! !!! ! !!! !! ! ! ! ! ! ! !


! !!!!!! !!! ! !!! ! ! ! ! !! ! ! !!! ! !! ! ! !!! ! !!!!!! !!! ! !!! ! ! ! ! !! ! ! !!! ! !! ! ! !!!

!! ! ! ! !! ! !! ! ! ! ! ! !! ! ! ! !! ! ! ! !!! ! ! !! ! ! ! !! ! !! ! ! ! ! ! ! ! !! ! ! !! ! !!! !
!!! ! ! ! !! ! ! ! !!! ! ! !!!
!! ! !! ! ! !! ! !! !
ID: e470e19d

ID: 6e6a3241
The function is defined by . What is the x-intercept of the graph of in the xy-plane?

A.

B.
Which of the following graphs in the xy-plane could be used to
C. solve the system of equations above?

D.
A.

ID: e470e19d Answer


Correct Answer: D

Rationale
B.
Choice D is correct. The given function 𝑓 is a linear function. Therefore, the graph of 𝑦 = 𝑓(𝑥) in the xy-plane
has one x-intercept at the point (𝑘, 0), where 𝑘 is a constant. Substituting 0 for 𝑓(𝑥) and 𝑘 for 𝑥 in the given
function yields 0 = 7𝑘 − 84. Adding 84 to both sides of this equation yields 84 = 7𝑘. Dividing both sides of this
equation by 7 yields 12 = 𝑘. Therefore, the x-intercept of the graph of 𝑦 = 𝑓(𝑥) in the xy-plane is (12, 0).
C.
Choice A is incorrect and may result from conceptual or calculation errors.

Choice B is incorrect and may result from conceptual or calculation errors.

Choice C is incorrect and may result from conceptual or calculation errors. D.

Question Difficulty: Medium

ID: 6e6a3241 Answer


Correct Answer: C

Rationale

Choice C is correct. The graph of a system of equations is the graph that shows the lines represented by each
of the equations in the system. The x-intercept of the graph of each given equation can be found by
substituting 0 for y in each equation: , or , and , or . The y-intercept of the
graph of each equation can be found by substituting 0 for x in each equation: , or , and
or . Using these x- and y- intercept values, the line that has equation passes

about:srcdoc Page 95 of 126 about:srcdoc Page 96 of 126


10/30/23, 02:04 10/30/23, 02:04

through the points and , and the line that has equation passes through the points
and . Only the lines in choice C pass through these points and can be used to solve the given system of ! ! ! ! !!! ! !!! !! ! ! ! ! ! ! !
equations.
! !!!!!! !!! ! !!! ! ! ! ! !! ! ! !!! ! !! ! ! !!!
Choices A, B, and D are incorrect. In choices A and B, neither line passes through and or and

. In choice D, although one line passes through and the other line doesn’t pass through !! ! ! ! !! ! !! ! ! ! ! ! !! ! ! ! !! ! ! ! !!! ! !
and .
ID: 620fe971
Question Difficulty: Medium
A team of workers has been moving cargo off of a ship. The equation below models the approximate
number of tons of cargo, y, that remains to be moved x hours after the team started working.

The graph of this equation in the xy-plane is a line. What is the best
interpretation of the x-intercept in this context?

A. The team will have moved all the cargo in about 4.8 hours.

B. The team has been moving about 4.8 tons of cargo per hour.

C. The team has been moving about 25 tons of cargo per hour.

D. The team started with 120 tons of cargo to move.

ID: 620fe971 Answer


Correct Answer: A

Rationale

Choice A is correct. The x-intercept of the line with equation y = 120 – 25x can be found by substituting 0 for y
and finding the value of x. When y = 0, x = 4.8, so the x-intercept is at (4.8, 0). Since y represents the number of
tons of cargo remaining to be moved x hours after the team started working, it follows that the x-intercept
refers to the team having no cargo remaining to be moved after 4.8 hours. In other words, the team will have
moved all of the cargo after about 4.8 hours.

Choice B is incorrect and may result from incorrectly interpreting the value 4.8. Choices C and D are incorrect
and may result from misunderstanding the x-intercept. These statements are accurate but not directly
relevant to the x-intercept.

Question Difficulty: Medium

about:srcdoc Page 97 of 126 about:srcdoc Page 98 of 126


10/30/23, 02:04 10/30/23, 02:04

! ! ! ! !!! ! !!! !! ! ! ! ! ! ! !
! !!!!!! !!! ! !!! ! ! ! ! !! ! ! !!! ! !! ! ! !!!

!! ! ! ! !! ! !! ! ! ! ! ! ! ! !! ! ! !! ! !!! !
!!! ! ! ! !! ! ! ! !!! ! ! !!!
!! ! !! ! ! !! ! !! !

ID: 6a87902f

At how many points do the graphs of the given equations intersect in the xy-plane?

A. Zero

B. Exactly one

C. Exactly two

D. Infinitely many

ID: 6a87902f Answer


Correct Answer: A

Rationale

Choice A is correct. A system of two linear equations in two variables, 𝑥 and 𝑦, has zero points of intersection if

xy-plane represented by equations in slope-intercept form, 𝑦 = 𝑚𝑥 + 𝑏, are distinct if the y-coordinates of their
the lines represented by the equations in the xy-plane are distinct and parallel. The graphs of two lines in the

y-intercepts, 𝑏, are different and are parallel if their slopes, 𝑚, are the same. For the two equations in the given
system, 𝑦 = 2𝑥 + 10 and 𝑦 = 2𝑥 − 1, the values of 𝑏 are 10 and −1, respectively, and the values of 𝑚 are both 2.
Since the values of 𝑏 are different, the graphs of these lines have different y-coordinates of the y-intercept and
are distinct. Since the values of 𝑚 are the same, the graphs of these lines have the same slope and are parallel.
Therefore, the graphs of the given equations are lines that intersect at zero points in the xy-plane.

Choice B is incorrect. The graphs of a system of two linear equations have exactly one point of intersection if
the lines represented by the equations have different slopes. Since the given equations represent lines with
the same slope, there is not exactly one intersection point.

Choice C is incorrect. The graphs of a system of two linear equations can never have exactly two intersection
points.

Choice D is incorrect. The graphs of a system of two linear equations have infinitely many intersection points
when the lines represented by the equations have the same slope and the same y-coordinate of the y-
intercept. Since the given equations represent lines with different y-coordinates of their y-intercepts, there are
not infinitely many intersection points.

Question Difficulty: Medium

about:srcdoc Page 99 of 126 about:srcdoc Page 100 of 126


10/30/23, 02:04 10/30/23, 02:04

! ! ! ! !!! ! !!! !! ! ! ! ! ! ! ! ! ! ! ! !!! ! !!! !! ! ! ! ! ! ! !


! !!!!!! !!! ! !!! ! ! ! ! !! ! ! !!! ! !! ! ! !!! ! !!!!!! !!! ! !!! ! ! ! ! !! ! ! !!! ! !! ! ! !!!

!! ! ! ! !! ! !! ! ! ! ! ! !! ! ! ! !! ! ! ! !!! ! ! !!! !! ! ! ! !! ! !! ! ! ! ! ! !! ! ! ! !! ! ! ! !!! ! ! !!!


! ! ! !! ! ! !! ! !! ! ! ! !! ! ! !! ! !!

ID: 12ee1edc ID: 15daa8d6

In the given equation, b is a constant. If the equation has no In the given equation, is a constant. If the equation has infinitely many solutions, what is the value of ?
solution, what is the value of b ?

A. 2 ID: 15daa8d6 Answer


Correct Answer: 2
B. 4
Rationale

The correct answer is 2. An equation with one variable, 𝑥, has infinitely many solutions only when both sides
C. 6

of the equation are equal for any defined value of 𝑥. It's given that 2𝑥 + 16 = 𝑎(𝑥 + 8), where 𝑎 is a constant. This
equation can be rewritten as 2(𝑥 + 8) = 𝑎(𝑥 + 8). If this equation has infinitely many solutions, then both sides
D. 10

of this equation are equal for any defined value of 𝑥. Both sides of this equation are equal for any defined value
ID: 12ee1edc Answer of 𝑥 when 2 = 𝑎. Therefore, if the equation has infinitely many solutions, the value of 𝑎 is 2.

Alternate approach: If the given equation, 2𝑥 + 16 = 𝑎(𝑥 + 8), has infinitely many solutions, then both sides of
Correct Answer: A

this equation are equal for any value of 𝑥. If 𝑥 = 0, then substituting 0 for 𝑥 in 2𝑥 + 16 = 𝑎(𝑥 + 8) yields
Rationale

2(0) + 16 = 𝑎(0 + 8), or 16 = 8𝑎. Dividing both sides of this equation by 8 yields 2 = 𝑎.
Choice A is correct. This equation has no solution when there is no value of x that produces a true statement.

Question Difficulty: Medium


Solving the given equation for x by dividing both sides by gives . When , the
right-hand side of this equation will be undefined, and the equation will have no solution. Therefore, when
, there is no value of x that satisfies the given equation.

Choices B, C, and D are incorrect. Substituting 4, 6, and 10 for b in the given equation yields exactly one
solution, rather than no solution, for x. For example, substituting 4 for b in the given equation yields
, or . Dividing both sides of by 2 yields . Similarly, if or , and

, respectively.

Question Difficulty: Medium

about:srcdoc Page 101 of 126 about:srcdoc Page 102 of 126


10/30/23, 02:04 10/30/23, 02:04

! ! ! ! !!! ! !!! !! ! ! ! ! ! ! ! ! ! ! ! !!! ! !!! !! ! ! ! ! ! ! !


! !!!!!! !!! ! !!! ! ! ! ! !! ! ! !!! ! !! ! ! !!! ! !!!!!! !!! ! !!! ! ! ! ! !! ! ! !!! ! !! ! ! !!!

!! ! ! ! !! ! !! ! ! ! ! ! !! ! ! ! !! ! ! ! !!! ! ! !!! !! ! ! ! !! ! !! ! ! ! ! ! !! ! ! ! !! ! ! ! !!! ! ! !!!


! ! ! !! ! ! !! ! !! !! ! !! ! ! !! ! !! !

ID: aa85b138 ID: 9f3cb472

Line in the xy-plane has a slope of and passes through the point . Which equation defines line ?
A tree had a height of 6 feet when it was planted. The equation above can A.
be used to find how many years n it took the tree to reach a height of 14
feet. Which of the following is the best interpretation of the number 2 in this B.
context?
C.
A. The number of years it took the tree to double its height
D.
B. The average number of feet that the tree grew per year

C. The height, in feet, of the tree when the tree was 1 year old ID: 9f3cb472 Answer
D. The average number of years it takes similar trees to grow 14 feet Correct Answer: D

Rationale

Choice D is correct. The equation that defines line 𝑡 in the xy-plane can be written in slope-intercept form
1
ID: aa85b138 Answer
𝑦 = 𝑚𝑥 + 𝑏, where 𝑚 is the slope of line 𝑡 and (0, 𝑏) is its y-intercept. It’s given that line 𝑡 has a slope of − .
3
1 1 1 𝑥
Correct Answer: B
Therefore, 𝑚 = − . Substituting − for 𝑚 in the equation 𝑦 = 𝑚𝑥 + 𝑏 yields 𝑦 = − 𝑥 + 𝑏, or 𝑦 = − + 𝑏. It’s also
3 3 3 3
𝑥
given that line 𝑡 passes through the point (9, 10). Substituting 9 for 𝑥 and 10 for 𝑦 in the equation 𝑦 = − + 𝑏
Rationale

3
9
yields 10 = − + 𝑏, or 10 = − 3 + 𝑏. Adding 3 to both sides of this equation yields 13 = 𝑏. Substituting 13 for 𝑏 in
3
Choice B is correct. The height of the tree at a given time is equal to its height when it was planted plus the
𝑥 𝑥
the equation 𝑦 = − + 𝑏 yields 𝑦 = − + 13.
number of feet that the tree grew. In the given equation, 14 represents the height of the tree at the given time,
and 6 represents the height of the tree when it was planted. It follows that represents the number of feet 3 3
the tree grew from the time it was planted until the time it reached a height of 14 feet. Since n represents the Choice A is incorrect and may result from conceptual or calculation errors.

1
number of years between the given time and the time the tree was planted, 2 must represent the average
Choice B is incorrect. This equation defines a line that has a slope of 9, not − , and passes through the point
3
number of feet the tree grew each year.
(0, 10), not (9, 10).
Choice A is incorrect and may result from interpreting the coefficient 2 as doubling instead of as increasing by
2 each year. Choice C is incorrect. The height of the tree when it was 1 year old was feet, not 2 feet. Choice C is incorrect. This equation defines a line that passes through the point (0, 10), not (9, 10).
Choice D is incorrect. No information is given to connect the growth of one particular tree to the growth of
similar trees. Question Difficulty: Medium

Question Difficulty: Medium

about:srcdoc Page 103 of 126 about:srcdoc Page 104 of 126


10/30/23, 02:04 10/30/23, 02:04

! ! ! ! !!! ! !!! !! ! ! ! ! ! ! ! ! ! ! ! !!! ! !!! !! ! ! ! ! ! ! !


! !!!!!! !!! ! !!! ! ! ! ! !! ! ! !!! ! !! ! ! !!! ! !!!!!! !!! ! !!! ! ! ! ! !! ! ! !!! ! !! ! ! !!!

!! ! ! ! !! ! !! ! ! ! ! ! !! ! ! ! !!! ! ! ! ! !!!!! ! !! ! ! ! !! ! !! ! ! ! ! ! !! ! ! ! !! ! ! ! !!! ! ! !!!


!! !! ! ! !! ! !!! ! ! ! ! !! ! ! !! ! !!
! ! ! !! ! !! !

ID: 7a5a74a6
ID: 968e9e51

If x is the solution to the equation above,


what is the value of ?
Which of the following ordered pairs is a solution to
the system of inequalities above? A.

A.
B.
B.

C. C.

D.
D.

ID: 968e9e51 Answer


Correct Answer: D
ID: 7a5a74a6 Answer
Correct Answer: B
Rationale
Rationale
Choice D is correct. The solutions to the given system of inequalities is the set of all ordered pairs that
Choice B is correct. Because 2 is a factor of both and 6, the expression can be rewritten as .
satisfy both inequalities in the system. For an ordered pair to satisfy the inequality , the value of the
Substituting for on the left-hand side of the given equation yields
ordered pair’s y-coordinate must be less than or equal to the value of the ordered pair’s x-coordinate. This is
true of the ordered pair , because . To satisfy the inequality , the value of the ordered , or . Subtracting from both sides of this

pair’s y-coordinate must be less than or equal to the value of the additive inverse of the ordered pair’s x- equation yields . Adding 11 to both sides of this equation yields . Dividing both
coordinate. This is also true of the ordered pair . Because 0 is its own additive inverse, is the
same as . Therefore, the ordered pair is a solution to the given system of inequalities. sides of this equation by 2 yields .

Choice A is incorrect. This ordered pair satisfies only the inequality in the given system, not both Alternate approach: Distributing 3 to the quantity on the left-hand side of the given equation and

inequalities. Choice B incorrect. This ordered pair satisfies only the inequality in the system, but not distributing 4 to the quantity on the right-hand side yields , or

both inequalities. Choice C is incorrect. This ordered pair satisfies neither inequality. . Subtracting from both sides of this equation yields . Adding 29 to both sides

of this equation yields . Dividing both sides of this equation by 2 yields . Therefore, the value of

Question Difficulty: Medium


is , or .

about:srcdoc Page 105 of 126 about:srcdoc Page 106 of 126


10/30/23, 02:04 10/30/23, 02:04

Choice A is incorrect. This is the value of x, not . Choices C and D are incorrect. If the value of is

! ! ! ! !!! ! !!! !! ! ! ! ! ! ! !
or , it follows that the value of x is or , respectively. However, solving the given equation

! !!!!!! !!! ! !!! ! ! ! ! !! ! ! !!! ! !! ! ! !!!


for x yields . Therefore, the value of can’t be or .
!! ! ! ! !! ! !! ! ! ! ! ! !! ! ! ! !!! ! ! ! ! !!!!! !
Question Difficulty: Medium !! !! ! ! !! ! !!! !
! ! ! !! ! !! !

ID: 64c85440

In North America, the standard width of a parking space is at least 7.5 feet
and no more than 9.0 feet. A restaurant owner recently resurfaced the
restaurant’s parking lot and wants to determine the number of parking
spaces, n, in the parking lot that could be placed perpendicular to a curb
that is 135 feet long, based on the standard width of a parking space.
Which of the following describes all the possible values of n ?

A.

B.

C.

D.

ID: 64c85440 Answer


Correct Answer: D

Rationale

Choice D is correct. Placing the parking spaces with the minimum width of 7.5 feet gives the maximum
possible number of parking spaces. Thus, the maximum number that can be placed perpendicular to a 135-

foot-long curb is . Placing the parking spaces with the maximum width of 9 feet gives the minimum
number of parking spaces. Thus, the minimum number that can be placed perpendicular to a 135-foot-long

curb is . Therefore, if n is the number of parking spaces in the lot, the range of possible values for n
is .

Choices A and C are incorrect. These choices equate the length of the curb with the maximum possible
number of parking spaces. Choice B is incorrect. This is the range of possible values for the width of a parking
space instead of the range of possible values for the number of parking spaces.

Question Difficulty: Medium

about:srcdoc Page 107 of 126 about:srcdoc Page 108 of 126


10/30/23, 02:04 10/30/23, 02:04

number of people who slept in tents would be ; however, the total number of

! ! ! ! !!! ! !!! !! ! ! ! ! ! ! ! people who slept in tents was 202, not 200. Choice D is incorrect. If 18 tents were 2-person tents, then the
remaining 42 tents would be 4-person tents. Since all the tents were filled to capacity, the total number of
! !!!!!! !!! ! !!! ! ! ! ! !! ! ! !!! ! !! ! ! !!! people who slept in tents would be ; however, the total number of people who
slept in tents was 202, not 204.
!! ! ! ! !! ! !! ! ! ! ! ! ! ! !! ! ! !! ! !!! !
!!! ! ! ! !! ! ! ! !!! ! ! !!! Question Difficulty: Medium
!! ! !! ! ! !! ! !! !

ID: 71189542

A group of 202 people went on an overnight camping trip, taking 60 tents


with them. Some of the tents held 2 people each, and the rest held 4 people
each. Assuming all the tents were filled to capacity and every person got to
sleep in a tent, exactly how many of the tents were 2-person tents?

A. 30

B. 20

C. 19

D. 18

ID: 71189542 Answer


Correct Answer: C

Rationale

Choice C is correct. Let x represent the number of 2-person tents and let y represent the number of 4-person
tents. It is given that the total number of tents was 60 and the total number of people in the group was 202.
This situation can be expressed as a system of two equations, and . The first equation
can be rewritten as . Substituting for y in the equation yields
. Distributing and combining like terms gives . Subtracting 240 from

both sides of and then dividing both sides by gives . Therefore, the number of 2-
person tents is 19.

Alternate approach: If each of the 60 tents held 4 people, the total number of people that could be
accommodated in tents would be 240. However, the actual number of people who slept in tents was 202. The
difference of 38 accounts for the 2-person tents. Since each of these tents holds 2 people fewer than a 4-person

tent, gives the number of 2-person tents.

Choice A is incorrect. This choice may result from assuming exactly half of the tents hold 2 people. If that
were true, then the total number of people who slept in tents would be ; however, the total
number of people who slept in tents was 202, not 180. Choice B is incorrect. If 20 tents were 2-person tents,

then the remaining 40 tents would be 4-person tents. Since all the tents were filled to capacity, the total

about:srcdoc Page 109 of 126 about:srcdoc Page 110 of 126


10/30/23, 02:04 10/30/23, 02:04

! ! ! ! !!! ! !!! !! ! ! ! ! ! ! ! ! ! ! ! !!! ! !!! !! ! ! ! ! ! ! !


! !!!!!! !!! ! !!! ! ! ! ! !! ! ! !!! ! !! ! ! !!! ! !!!!!! !!! ! !!! ! ! ! ! !! ! ! !!! ! !! ! ! !!!

!! ! ! ! !! ! !! ! ! ! ! ! !! ! ! ! !! ! ! ! !!! ! ! !! ! ! ! !! ! !! ! ! ! ! ! ! ! !! ! ! !! ! !!! !
!!! ! ! ! !! ! ! ! !!! ! ! !!!
!! ! !! ! ! !! ! !! !
ID: c1bd5301

ID: 7efe5495
A model predicts that a certain animal weighed pounds when it was born and that the animal gained
pounds per day in its first year of life. This model is defined by an equation in the form , where
is the predicted weight, in pounds, of the animal days after it was born, and and are constants. What is
the value of ?
The solution to the given system of equations is . What is the value of ?

A.
ID: c1bd5301 Answer
Correct Answer: 241 B.

Rationale C.

The correct answer is 241. For a certain animal, it's given that a model predicts the animal weighed 241 pounds
when it was born and gained 3 pounds per day in its first year of life. It's also given that this model is defined
D.

by an equation in the form 𝑓(𝑥) = 𝑎 + 𝑏𝑥, where 𝑓(𝑥) is the predicted weight, in pounds, of the animal 𝑥 days
after it was born, and 𝑎 and 𝑏 are constants. It follows that 𝑎 represents the predicted weight, in pounds, of the ID: 7efe5495 Answer
animal when it was born and 𝑏 represents the predicted rate of weight gain, in pounds per day, in its first year
of life. Thus, the value of 𝑎 is 241.
Correct Answer: C

Rationale

Choice C is correct. It's given by the first equation in the system that 𝑦 = 3𝑥. Substituting 3𝑥 for 𝑦 in the
Question Difficulty: Medium

equation 2𝑥 + 𝑦 = 12 yields 2𝑥 + 3𝑥 = 12, or 5𝑥 = 12.

Choice A is incorrect and may result from conceptual or calculation errors.

Choice B is incorrect and may result from conceptual or calculation errors.

Choice D is incorrect and may result from conceptual or calculation errors.

Question Difficulty: Medium

about:srcdoc Page 111 of 126 about:srcdoc Page 112 of 126


10/30/23, 02:04 10/30/23, 02:04

senior director, it follows that and . All four of these conditions are represented correctly in choice
! ! ! ! !!! ! !!! !! ! ! ! ! ! ! ! B.

! !!!!!! !!! ! !!! ! ! ! ! !! ! ! !!! ! !! ! ! !!! Choices A and C are incorrect. For example, the first condition, , in each of these options
implies that Marisa can pay the new staff members more than her budget of $9,700. Choice D is incorrect
!! ! ! ! !! ! !! ! ! ! ! ! !! ! ! ! !!! ! ! ! ! !!!!! ! because Marisa needs to hire at least 10 staff members, not at most 10 staff members, as the inequality
!! !! ! ! !! ! !!! ! implies.
! ! ! !! ! !! !

Question Difficulty: Medium


ID: b1228811

Marisa needs to hire at least 10 staff members for an upcoming project. The
staff members will be made up of junior directors, who will be paid
$640 per week, and senior directors, who will be paid $880 per week. Her
budget for paying the staff members is no more than $9,700 per week. She
must hire at least 3 junior directors and at least 1 senior director. Which of
the following systems of inequalities represents the conditions described if
x is the number of junior directors and y is the number of senior directors?

A.

B.

C.

D.

ID: b1228811 Answer


Correct Answer: B

Rationale

Choice B is correct. Marisa will hire x junior directors and y senior directors. Since she needs to hire at least 10
staff members, . Each junior director will be paid $640 per week, and each senior director will be

paid $880 per week. Marisa’s budget for paying the new staff is no more than $9,700 per week; in terms of x
and y, this condition is . Since Marisa must hire at least 3 junior directors and at least 1

about:srcdoc Page 113 of 126 about:srcdoc Page 114 of 126


10/30/23, 02:04 10/30/23, 02:04

! ! ! ! !!! ! !!! !! ! ! ! ! ! ! ! ! ! ! ! !!! ! !!! !! ! ! ! ! ! ! !


! !!!!!! !!! ! !!! ! ! ! ! !! ! ! !!! ! !! ! ! !!! ! !!!!!! !!! ! !!! ! ! ! ! !! ! ! !!! ! !! ! ! !!!

!! ! ! ! !! ! !! ! ! ! ! ! ! ! !! ! ! !! ! !!! ! !! ! ! ! !! ! !! ! ! ! ! ! !! ! ! ! !! ! ! ! !!! ! !
!!! ! ! ! !! ! ! ! !!! ! ! !!!
!! ! !! ! ! !! ! !! !
ID: 7e3f8363

ID: 0dd6227f
In the xy-plane, the graph of the linear function contains the points and . Which equation defines
, where ?
At how many points do the graphs of the equations and intersect in the xy-plane?
A.
A.
B.
B.
C.
C.
D.
D.

ID: 7e3f8363 Answer


ID: 0dd6227f Answer
Correct Answer: C
Correct Answer: B
Rationale
Rationale

Choice B is correct. Each given equation is written in slope-intercept form, 𝑦 = 𝑚𝑥 + 𝑏, where 𝑚 is the slope
𝑓(𝑥) = 𝑚𝑥 + 𝑏, where 𝑚 represents the slope and (0, 𝑏) represents the y-intercept of the graph of 𝑦 = 𝑓(𝑥). It’s
Choice C is correct. In the xy-plane, an equation of the graph of a linear function can be written in the form

and (0, 𝑏) is the y-intercept of the graph of the equation in the xy-plane. The graphs of two lines that have
given that the graph of the linear function 𝑓, where 𝑦 = 𝑓(𝑥), in the xy-plane contains the point (0, 3). Thus,
different slopes will intersect at exactly one point. The graph of the first equation is a line with slope 1. The
𝑏 = 3. The slope of the graph of a line containing any two points (𝑥1 , 𝑦1 ) and (𝑥2 , 𝑦2 ) can be found using the
graph of the second equation is a line with slope 8. Since the graphs are lines with different slopes, they will 𝑦 − 𝑦1
slope formula, 𝑚 = 2 . Since it’s given that the graph of the linear function 𝑓 contains the points (0, 3) and
𝑥2 − 𝑥1
31 − 3
(7, 31), it follows that the slope of the graph of the line containing these points is 𝑚 = , or 𝑚 = 4.
intersect at exactly one point.

7−0
Choice A is incorrect because two graphs of linear equations have 0 intersection points only if they are parallel ( ) ( )
Substituting 4 for 𝑚 and 3 for 𝑏 in 𝑓 𝑥 = 𝑚𝑥 + 𝑏 yields 𝑓 𝑥 = 4𝑥 + 3.
and therefore have the same slope.
Choice A is incorrect. This function represents a graph with a slope of 28 and a y-intercept of (0, 34).
Choice C is incorrect because two graphs of linear equations in the xy-plane can have only 0, 1, or infinitely
many points of intersection. Choice B is incorrect. This function represents a graph with a slope of 3 and a y-intercept of (0, 38).

Choice D is incorrect because two graphs of linear equations in the xy-plane can have only 0, 1, or infinitely Choice D is incorrect. This function represents a graph with a slope of 7 and a y-intercept of (0, 3).
many points of intersection.
Question Difficulty: Medium
Question Difficulty: Medium

about:srcdoc Page 115 of 126 about:srcdoc Page 116 of 126


10/30/23, 02:04 10/30/23, 02:04

! ! ! ! !!! ! !!! !! ! ! ! ! ! ! ! ! ! ! ! !!! ! !!! !! ! ! ! ! ! ! !


! !!!!!! !!! ! !!! ! ! ! ! !! ! ! !!! ! !! ! ! !!! ! !!!!!! !!! ! !!! ! ! ! ! !! ! ! !!! ! !! ! ! !!!

!! ! ! ! !! ! !! ! ! ! ! ! ! ! !! ! ! !! ! !!! ! !! ! ! ! !! ! !! ! ! ! ! ! !! ! ! ! !! ! ! ! !!! ! !
!!! ! ! ! !! ! ! ! !!! ! ! !!!
!! ! !! ! ! !! ! !! !
ID: a5834ea4
ID: bf4a8b6a

For the given linear function , which table gives three values of and their corresponding values of ?
A company that provides whale-watching tours takes groups of people at a time. The company’s revenue is
dollars per adult and dollars per child. If the company’s revenue for one group consisting of adults and children A.
was dollars, how many people in the group were children?

A.

B.

C.

D.
B.

ID: bf4a8b6a Answer


Correct Answer: C

Rationale

Choice C is correct. Let 𝑥 represent the number of children in a whale-watching tour group. Let 𝑦 represent the
number of adults in this group. Because it's given that 21 people are in a group and the group consists of adults
and children, it must be true that 𝑥 + 𝑦 = 21. Since the company's revenue is 60 dollars per child, the total
C.

revenue from 𝑥 children in this group was 60𝑥 dollars. Since the company's revenue is 80 dollars per adult, the
total revenue from 𝑦 adults in this group was 80𝑦 dollars. Because it's given that the total revenue for this group
was 1,440 dollars, it must be true that 60𝑥 + 80𝑦 = 1,440. The equations 𝑥 + 𝑦 = 21 and 60𝑥 + 80𝑦 = 1,440 form a
linear system of equations that can be solved to find the value of 𝑥, which represents the number of children in
the group, using the elimination method. Multiplying both sides of the equation 𝑥 + 𝑦 = 21 by 80 yields
80𝑥 + 80𝑦 = 1,680. Subtracting 60𝑥 + 80𝑦 = 1,440 from 80𝑥 + 80𝑦 = 1,680 yields
(80𝑥 + 80𝑦) − (60𝑥 + 80𝑦) = 1,680 − 1,440, which is equivalent to 80𝑥 − 60𝑥 + 80𝑦 − 80𝑦 = 240, or 20𝑥 = 240. Dividing
both sides of this equation by 20 yields 𝑥 = 12. Therefore, 12 people in the group were children.
D.

Choice A is incorrect and may result from conceptual or calculation errors.

Choice B is incorrect. This is the number of adults in the group, not the number of children in the group.

Choice D is incorrect and may result from conceptual or calculation errors.

Question Difficulty: Medium


ID: a5834ea4 Answer

Correct Answer: B

about:srcdoc Page 117 of 126 about:srcdoc Page 118 of 126


10/30/23, 02:04 10/30/23, 02:04

Rationale
! ! ! ! !!! ! !!! !! ! ! ! ! ! ! !
Choice B is correct. For the given linear function 𝑓, 𝑓(𝑥) must equal 39 for all values of 𝑥. Of the given choices,
only choice B gives three values of 𝑥 and their corresponding values of 𝑓(𝑥) for the given linear function 𝑓.
! !!!!!! !!! ! !!! ! ! ! ! !! ! ! !!! ! !! ! ! !!!
Choice A is incorrect and may result from conceptual errors.
!! ! ! ! !! ! !! ! ! ! ! ! !! ! ! ! !! ! ! ! !!! ! ! !!!
Choice C is incorrect and may result from conceptual errors. !! ! !! ! ! !! ! !! !

Choice D is incorrect and may result from conceptual errors.


ID: 01682aa5
Question Difficulty: Medium
Line is defined by . Line is perpendicular to line in the xy-plane. What is the slope of line ?

A.

B.

C.

D.

ID: 01682aa5 Answer


Correct Answer: C

Rationale

Choice C is correct. It’s given that line 𝑟 is perpendicular to line 𝑝 in the xy-plane. This means that the slope of
line 𝑟 is the negative reciprocal of the slope of line 𝑝. If the equation for line 𝑝 is rewritten in slope-intercept
form 𝑦 = 𝑚𝑥 + 𝑏, where 𝑚 and 𝑏 are constants, then 𝑚 is the slope of the line and (0, 𝑏) is its y-intercept.
Subtracting 18𝑥 from both sides of the equation 2𝑦 + 18𝑥 = 9 yields 2𝑦 = − 18𝑥 + 9. Dividing both sides of this
9
equation by 2 yields 𝑦 = − 9𝑥 + . It follows that the slope of line 𝑝 is −9. The negative reciprocal of a number is
2
−1 1 1
−1 divided by the number. Therefore, the negative reciprocal of −9 is , or . Thus, the slope of line 𝑟 is .
−9 9 9

Choice A is incorrect. This is the slope of line 𝑝, not line 𝑟.

Choice B is incorrect. This is the reciprocal, not the negative reciprocal, of the slope of line 𝑝.

Choice D is incorrect. This is the negative, not the negative reciprocal, of the slope of line 𝑝.

Question Difficulty: Medium

about:srcdoc Page 119 of 126 about:srcdoc Page 120 of 126


10/30/23, 02:04 10/30/23, 02:04

! ! ! ! !!! ! !!! !! ! ! ! ! ! ! ! ! ! ! ! !!! ! !!! !! ! ! ! ! ! ! !


! !!!!!! !!! ! !!! ! ! ! ! !! ! ! !!! ! !! ! ! !!! ! !!!!!! !!! ! !!! ! ! ! ! !! ! ! !!! ! !! ! ! !!!

!! ! ! ! !! ! !! ! ! ! ! ! !! ! ! ! !! ! ! ! !!! ! ! !! ! ! ! !! ! !! ! ! ! ! ! !! ! ! ! !! ! ! ! !!! ! !

ID: e62cfe5f ID: 548a4929

According to a model, the head width, in millimeters, of a worker bumblebee The function is defined by . The graph of in the xy-plane has an x-intercept at
can be estimated by adding 0.6 to four times the body weight of the bee, in and a y-intercept at , where and are constants. What is the value of ?
grams. According to the model, what would be the head width, in
A.
millimeters, of a worker bumblebee that has a body weight of 0.5 grams?
B.

ID: e62cfe5f Answer C.

Rationale D.

The correct answer is 2.6. According to the model, the head width, in millimeters, of a worker bumblebee can
be estimated by adding 0.6 to 4 times the body weight, in grams, of the bee. Let x represent the body weight, in
ID: 548a4929 Answer
grams, of a worker bumblebee and let y represent the head width, in millimeters. Translating the verbal
description of the model into an equation yields . Substituting 0.5 grams for x in this equation Correct Answer: A

yields , or . Therefore, a worker bumblebee with a body weight of 0.5 grams has an Rationale

Choice A is correct. The x-intercept of a graph in the xy-plane is the point on the graph where 𝑦 = 0. It's given
that function ℎ is defined by ℎ(𝑥) = 4𝑥 + 28. Therefore, the equation representing the graph of 𝑦 = ℎ(𝑥) is
estimated head width of 2.6 millimeters. Note that 2.6 and 13/5 are examples of ways to enter a correct answer.

𝑦 = 4𝑥 + 28. Substituting 0 for 𝑦 in the equation 𝑦 = 4𝑥 + 28 yields 0 = 4𝑥 + 28. Subtracting 28 from both sides of
Question Difficulty: Medium

this equation yields − 28 = 4𝑥. Dividing both sides of this equation by 4 yields − 7 = 𝑥. Therefore, the x-
intercept of the graph of 𝑦 = ℎ(𝑥) in the xy-plane is (−7, 0). It's given that the x-intercept of the graph of 𝑦 = ℎ(𝑥)
is (𝑎, 0). Therefore, 𝑎 = − 7. The y-intercept of a graph in the xy-plane is the point on the graph where 𝑥 = 0.
Substituting 0 for 𝑥 in the equation 𝑦 = 4𝑥 + 28 yields 𝑦 = 4(0) + 28, or 𝑦 = 28. Therefore, the y-intercept of the
graph of 𝑦 = ℎ(𝑥) in the xy-plane is (0, 28). It's given that the y-intercept of the graph of 𝑦 = ℎ(𝑥) is (0, 𝑏).
Therefore, 𝑏 = 28. If 𝑎 = − 7 and 𝑏 = 28, then the value of 𝑎 + 𝑏 is −7 + 28, or 21.

Choice B is incorrect. This is the value of 𝑏, not 𝑎 + 𝑏.

Choice C is incorrect and may result from conceptual or calculation errors.

Choice D is incorrect. This is the value of − 𝑎 + 𝑏, not 𝑎 + 𝑏.

Question Difficulty: Medium

about:srcdoc Page 121 of 126 about:srcdoc Page 122 of 126


10/30/23, 02:04 10/30/23, 02:04

from them. Therefore, .

! ! ! ! !!! ! !!! !! ! ! ! ! ! ! !
Choices A, B, and C are incorrect and may be the result of errors when solving the system.

! !!!!!! !!! ! !!! ! ! ! ! !! ! ! !!! ! !! ! ! !!!

!! ! ! ! !! ! !! ! ! ! ! ! ! ! !! ! ! !! ! !!! ! Question Difficulty: Medium


!!! ! ! ! !! ! ! ! !!! ! ! !!!
!! ! !! ! ! !! ! !! !

ID: cb8f449f

The system of equations above has solution (x,


y). What is the value of x ?

A.

B.

C.

D.

ID: cb8f449f Answer


Correct Answer: D

Rationale

Choice D is correct. Adding the corresponding sides of the two equations eliminates y and yields , as
shown.

If (x, y) is a solution to the system, then (x, y) satisfies both equations in the system and any equation derived

about:srcdoc Page 123 of 126 about:srcdoc Page 124 of 126


10/30/23, 02:04 10/30/23, 02:04

! ! ! ! !!! ! !!! !! ! ! ! ! ! ! ! ! ! ! ! !!! ! !!! !! ! ! ! ! ! ! !


! !!!!!! !!! ! !!! ! ! ! ! !! ! ! !!! ! !! ! ! !!! ! !!!!!! !!! ! !!! ! ! ! ! !! ! ! !!! ! !! ! ! !!!

!! ! ! ! !! ! !! ! ! ! ! ! !! ! ! ! !!! ! ! ! ! !!!!! ! !! ! ! ! !! ! !! ! ! ! ! ! !! ! ! ! !! ! ! ! !!! ! ! !!!


!! !! ! ! !! ! !!! ! !! ! !! ! ! !! ! !! !
! ! ! !! ! !! !

ID: 002dba45
ID: f224df07
Line is defined by . Line is perpendicular to line in the xy-plane. What is the slope of line ?
A cargo helicopter delivers only 100-pound packages and 120-pound
packages. For each delivery trip, the helicopter must carry at least 10
packages, and the total weight of the packages can be at most 1,100 ID: 002dba45 Answer
pounds. What is the maximum number of 120-pound packages that the Correct Answer: .1764, .1765, 3/17
helicopter can carry per trip?
Rationale

3
. It’s given that line 𝑗 is perpendicular to line 𝑘 in the xy-plane. This means that the
A. 2

17
17
The correct answer is
slope of line 𝑗 is the negative reciprocal of the slope of line 𝑘. The equation of line 𝑘, 𝑦 = − 𝑥 + 5, is written in
3
B. 4

slope-intercept form 𝑦 = 𝑚𝑥 + 𝑏, where 𝑚 is the slope of the line and 𝑏 is the y-coordinate of the y-intercept of
17
the line. It follows that the slope of line 𝑘 is − . The negative reciprocal of a number is −1 divided by the
C. 5

3
17 −1 3 3
number. Therefore, the negative reciprocal of − is 17, or . Thus, the slope of line 𝑗 is . Note that 3/17,
3 17 17
D. 6


3
ID: f224df07 Answer .1764, .1765, and 0.176 are examples of ways to enter a correct answer.

Correct Answer: C Question Difficulty: Medium


Rationale

Choice C is correct. Let a equal the number of 120-pound packages, and let b equal the number of 100-pound
packages. It’s given that the total weight of the packages can be at most 1,100 pounds: the inequality
represents this situation. It’s also given that the helicopter must carry at least 10
packages: the inequality represents this situation. Values of a and b that satisfy these two
inequalities represent the allowable numbers of 120-pound packages and 100-pound packages the helicopter
can transport. To maximize the number of 120-pound packages, a, in the helicopter, the number of 100-pound
packages, b, in the helicopter needs to be minimized. Expressing b in terms of a in the second inequality yields
, so the minimum value of b is equal to . Substituting for b in the first inequality results

in . Using the distributive property to rewrite this inequality yields


, or . Subtracting 1,000 from both sides of this inequality
yields . Dividing both sides of this inequality by 20 results in . This means that the maximum

number of 120-pound packages that the helicopter can carry per trip is 5.

Choices A, B, and D are incorrect and may result from incorrectly creating or solving the system of
inequalities.

Question Difficulty: Medium

about:srcdoc Page 125 of 126 about:srcdoc Page 126 of 126

You might also like